PPPJHH

Download as pdf or txt
Download as pdf or txt
You are on page 1of 126

KINEMATICS

(Raj Sinha Sir)


SESSION – 1 AND 2

AIM
 Basic definitions and their explanations

Distance: The actual path length of the particle from initial to final positions of journey.
Symbol: “S” SI unit: m
The distance travelled by the particle is independent of direction of motion and it is scalar quantity.

Displacement: The shortest distance between initial and final positions of journey.
Displacement is a vector quantity and its magnitude is the shortest distance between the initial and final
positions of journey.
Displacement and distance can be evaluated not only between initial and final positions but also between
any two points of journey.
Consider A and B are two positions of particle in its journey.

𝑂𝐴 = 𝑟
𝑂𝐵 = 𝑟
𝑂𝐴 = 𝑟
𝑟 →Final position vector
𝑟 → Initial position vector
𝑟 → Displacement vector
Hence displacement is “final position vector” minus “initial position vector”. Displacement can have zero
magnitude, but distance travelled can never be zero.
If r =a1𝚤̂+b1𝚥̂+c1𝑘 and r = a2𝚤̂+b2𝚥̂+c2𝑘 then r = r − r = (a2𝚤̂+b2𝚥̂+c2𝑘 ) – (a1𝚤̂+b1𝚥̂+c1𝑘)
𝑟 = (a2–a1) 𝚤̂+ (b2–b1) 𝚥̂+ (c2–c1)𝑘
Speed: The distance travelled per unit time
Symbol: “v” SI unit: ms-1
Speed is the scalar quantity
Velocity: The displacement of the particle per unit time.
Symbol: “v”: SI unit: ms-1

Raj Sinha Sir 62


Velocity is a vector quantity; its direction is same as that of displacement.
Let us consider an example to discuss their concepts:

A B

3
Let a particle move from point A to point B as shown:
Along path 1, distance travelled is 100m, time taken is 10s
Along path 2, distance is 200m, time taken is 15s
Along path 3, distance travelled is 150m, time taken is 12s

Along path 1: distance = s =100m: speed = ms–1 = 10 ms–1;

Velocity = 𝑣̅ = ms–1 (from A to B) ⟹ 𝑣̅ = 10ms–1 from A to B (displacement is same distance along


path 1)

Along path 2: displacement = 100 m; time = 15 s speed = = ms–1;

Velocity = 𝑣̅ = = ms–1 (from A to B)

Along path 3: displacement = 100 m; distance = 150m; time = 12s


Speed = = ms–1; velocity = = ms–1 (from A to B)

Note: The direction of displacement and velocity is same for all the three paths i.e. imagine a vector drawn
from A to B. The direction of vector gives the direction of displacement and velocity.
Average Speed: - When any object travels with non-uniform speeds during different time intervals or
travelling different segments of path, the ratio of total distance covered to the total time taken is called
'Average Speed'. e.g.
( )
Average Speed=
( )

If an object travels with speeds v1, v2, v3........vN, during time intervals t1, t2, t3 ...tN, travelling for
distances s1, s2, s3..........sN, its average speed can be discussed in following three ways.

63
When Distances & Speeds are When Distances & Time When Speeds & Time intervals
Mentioned intervals are mentioned are mentioned
Average Speed (𝑣̅ ) Average Speed (𝑣̅ ) Average Speed (𝑣̅ ) =
….
= + …. 𝑣 𝑡 + 𝑣 𝑡 …𝑣 𝑡
=
…. 𝑡 + 𝑡 …𝑡

If s1 = s2 = s3........ =sN= then If t1 = t2 = t3...........tn= then



Average Speed (𝑣̅ ) = Average Speed (𝑣̅ ) =

i.e. Harmonic Mean of Velocities i.e. Arithmetic Mean of Velocities


These are Basic independent cases. In practice you may have combination of two or more.
Average velocity: It is defined as the ratio of displacement to time taken by the body


Average Speed (𝑣̅ ) = ;𝑣 ⃗=

Instantaneous velocity:
The term “instantaneous” is used to explain an event happening in a very smallinterval of time. Usually
instantaneous time is denoted as “dt”. The numerical value of ‘dt” is notadefinite constant, but we can say
that it less than one and close to zero i.e. about 0.01s or 0.001s. (Let us not assign a fixed value for it. Its
depends on the context of explanation).
Let us consider a situation of a particle moving along a straight line from A to B. At a time t = 0 (usually
called initial time) the particle is at A and at a time t = 10s. Let AB = 100m. As the particle crosses the point
C, can we guess time taken to cross this point. The answer is simple. “Impossible”. If we consider two
nearest points, one point D just before reaching C and another point E just after crossing C. The time
interval from D to E is taken as instantaneous time i.e. “dt”. The displacement/ distance between D and E
are denoted as 𝑑𝑠/𝑑𝑠 i.e.
𝐷𝑆iscalled infinitesimal (very small) displacement (𝑑𝑠) DE is called infinitesimal distance (ds)
Instantaneous speed: It is the infinitesimal distance travelled per infinitesimal time.
𝑑𝑠
Vinst =
Instantaneous velocity: It is the ratio of infinitesimal displacement to infinitesimal time.
𝑑𝑠
𝑉 inst=
Note: Always it is assumed that the particle is along a straight line path in this infinitesimal time.

64
Uniform speed:Speed of the particle is said to be uniform, if it covers equal distances in equal intervals of
time taken, however small the intervals are.
i.e. VAB = VAc = VCB = VDE

Non-uniform Speed: Speed of the particle is said to be non-uniform, if it covers unequal distances in equal
intervals of time, however small the intervals are.

Non-uniform velocity: Velocity of the particle is said to be non-uniform, if it travels unequal displacements
in equal intervals of time, however small the intervals are.
Uniform speed is possible for any path of particle i.e. straight line, curved paths. Uniform velocity is
possible only for a straight line path, because velocity is a vector quantity i.e. for velocity to be constant, the
direction of velocity should be same.

CLASS EXERCISE SOLVED:


Problem1.A man goes 10m towards North, and then 20m towards east then displacement is
a) 22.5m b) 25m c) 25.5m d) 30m
Solution: (a)
If we take east as axis and north as axis, then displacement = 20𝚤̂+10𝚥̂
So, magnitude of displacement = √𝟐𝟎𝟐 + 𝟏𝟎𝟐 = 𝟏𝟎√𝟓 = 22.5𝑚

Problem2. A body moves over one fourth of a circular arc in a circle of radius r. The magnitude of distance
travelled and displacement will be respectively
a) ,𝑟√2 b) ,𝑟 c) 𝜋r, d) 𝜋r, r

Solution: (a) Let particle start from A, its position vector 𝑟⃗OA= r𝚤̂.After one quarter position vector
𝑟⃗OB= r𝚥̂.
So displacement = r𝚥̂ − r𝚤̂Magnitude of displacement = r√2 and distance = one fourth of circumference =
2𝜋𝑟
=

Y
B

X
O A

65
Problem3. The displacement of the point of the wheel initially in contact with the ground, when the wheel
roles forward half a revolution will be (radius of the wheel is R)
a) b)R√𝜋 + 4 c) 2𝜋R d) 𝜋R

Solution: (b)
Pnew

2R

Pinitial
R

Horizontal distance covered by the wheel in half revolution = 𝜋R so the displacement of the point which
was initially in contact with a ground = (𝜋𝑅) + (2𝑅) = R√𝜋 + 4.

Problem4. If a car covers 2/5th of the total distance with v1 speed and 3/5th distance with v2 then average
speed is
a) √𝑣 𝑣 b) c) d)
3𝑣
Solution:

(d) Average speed = = ( / ) ( / ) =


𝑡𝑖𝑚𝑒

(2/5)x (3/5)x
t1 t2

Problem5. A car accelerated from initial position and then returned at initial point, then
a) Velocity is zero but speed increases b) Speed is zero but velocity increases
c) Both speed and velocity increase d) Both speed and velocity decrease
Solution: (a) As the net displacement = 0 Hence velocity = 0; but speed increases.

| |
Note:
|𝐴𝑣𝑒𝑟𝑎𝑔𝑒 |
 1 ⟹ |𝐴. 𝑠𝑝𝑒𝑒𝑑| |𝐴𝑣. 𝑣𝑒𝑙𝑜𝑐𝑖𝑡𝑦|

66
Problem 6. A man walks on a straight road from his home to a market 2.5 km away with a speed of
5 km/h. Finding the market closed, he instantly turns and walks back home with a speed of 7.5 km/h. The
average speed of the man over the interval of time 0 to 40 min. is equal to
a) 5 km/h b) km/h c) km/h d) km/h
.
Solution: (d) Time taken in going to market = = hr = 30 min. As we told to find average speed for the
interval 40 min, so remaining time for consideration of motion is 10 min. So distance travelled in remaining
time for consideration of motion is 10 min. So distance travelled in remaining
10 min = 7.5 × = 1.25 km.
( . . )
Hence, average speed = = = km/hr.

Problem7. The relation 3t=√3𝑥 + 6describes the displacement of a particle in one direction where is in
metres and t in sec. The displacement, when velocity is zero, is
a) 24 metres b) 12 metres c) 5 metres d) Zero
Solution: (d)3t = √3x+6 ⟹ √3𝑥 = (3𝑡 − 6) ⟹3x = (3t–6)2⟹x = 3t2–12t+12

∴v= = (3𝑡 − 12𝑡 + 12 ) = 6𝑡 − 12


If velocity = 0 then, ⟹ 𝑡 = 2𝑠𝑒𝑐
Hence at t = 2, x = 3(2)2 - 12 (2) + 12 = 0 metres.

Problem8. The motion of a particle is described by the equation where cm and cm. Its instantaneous
velocity at time 3 sec will be
a) 36 cm/sec b) 18 cm/sec c) 16 cm/sec d) 32 cm/sec
Solution: (b) x = a +bt2 ∴v= = 0 + 2𝑏𝑡
At t = 3sec, v = 2×3×3 = 18 cm/sec (As b = 3 cm)

Problem9. A train has a speed of 60 km/h for the first one hour and 40 km/h for the next half hour. Its
average speed in km/h is
a) 50 b) 53.33 c) 48 d) 70
Solution: (b) Total distance travelled = 60×1+40× = 80 km and

Total time taken = 1 hr + hr = hr


80
∴ Average speed = = 53.33km/ h
/

67
Problem 10.A car moving on a straight road covers one third of the distance with 20 km/hr and the rest with
60 km/hr. The average speed is
a) 40 km/hr b) 80 km/hr c) 46 km/hr d) 36 km/hr
/ /
Solution: (d) Let total distance travelled = x and total time taken t1 + t2 = +

∴ Average speed = ( / ) ( / ) = = 36 km/ hr

CLASS EXERCISE
1] A circular park has a radius of 1 km. A man standing from the center of park, walking towards East and
going along the circumference of the park reaches the North gate. What is his displacement?
a)√2 km b) 1.57 km c) 1 km d) 2.57 km
2] If a cyclist takes one minute to complete half revolution on a circular path 120m radius, what is the
average velocity?
a) 1 m/s b) 2 m/s c) 3 m/s d) 4 m/s
3] A car covers the 1st half of the distance between two places at a speed of 40 km/hr and the second half
at 60 km/hr. The average speed of the car is
a) 100 km/hr b) 48 km/hr c) 50 km/hr d) 25 km/hr
4] An aeroplane moves 400m towards north, 300m towards west and then 1200m vertically upwards. Then
its displacement from the initial position is
a) 1300m b) 1400m c) 1500m d) 1600m
5] An ant starts from one corner of a cube of side length 3m and reaches the diagonally opposite corner.
The displacement is

HOME EXERCISE
1] A body moves along a circular track of radius ‘r’. It starts from one end of a diameter, moves along the
circular track and completes 1 revolutions. What is the ratio of the distance travelled by the body to its
displacement?
a) 3𝜋/2 b) 2/ 𝜋 c) 2 𝜋 d)1/2 𝜋
2] A bus travels the first one third distances at a speed of 10 kmph. The next one third distance at a speed
of 20 kmph and the last one third distance at a speed of 60 kmph. The average speed of the bus is
a) 16kmph b) 18kmph c) 9kmph d) 48 kmph
3] A body moving in a straight line covers half the distance with a speed V, the remaining part of the
distance was covered with a speed V1 for half the time and with a speed V2 for other half of the time.
What is the average speed of the body?
( ) ( )
a) b) c) d)
( ) ( ) ( ) ( )

68
4] A particle moves along the side of a square of length ‘l’ starting from A and reaches the opposite corner
C by travelling from A to B and from B to C. If the time taken is‘t’, the average velocity of the particle
is l
B C

l l

A D
l

a) b) c) zero d)

5] A particle travels 3km west and 3km south, the displacement is
a) 3√2 km Sw b)3√2km Nw c) 3√3km NE d)3√2 km Nw
6] A particle travels 5km West and then 3 km East, the displacement is
a) 2kmWest b) 2 km East c) 1 km South d) 2 km North
7] A particle moves in a circular path of radius 5m with a time period of 5s. The distance, displacement,
average speed, average velocity after 2seconds is
8] A particle constrained to move on a straight line path. It returns to the starting point after 10sec. The
total distance covered by the particle during this time is 30m. Which of the following statements about
the motion of the particle is false?
A) displacement of the particle is zero
B) average speed of the particle is 3 ms-1
C) displacement of the particle is 30m
D) both a & b
a) A is correct b) B is correct c) C is correct d) D is correct

69
SESSION – 3, 4 AND 5

AIM
 Definition of acceleration, explanation of equations of motion.

Acceleration:The rate of change of velocity is acceleration.


The direction of acceleration is the direction of change in velocity
There is misconception that direction of acceleration is either along the direction of motion or opposite
to direction of motion. It need not be parallel to direction of motion.

𝑎= ; 𝑉 1 and 𝑉 2 need not be in the same direction 𝑉 2–𝑉 1need not be either in the direction of
𝑉 1 or 𝑉 2.

For a particle moving along a straight line, direction of acceleration is either in the direction of motion
(in case of increasing velocity) or in the opposite to direction of motion (in case of decreasing velocity)
Uniform acceleration: The body is said possess uniform acceleration if it executes equal change in velocity
in equal intervals of time, however small the intervals are.
There are two possible paths for this type of motion (i) straight line (ii) parabolic
Non–uniform acceleration: The body is said to possess non–uniform acceleration, if it executes unequal
change in velocity in equal intervals of time, however small the intervals are.
Average Acceleration: –If Velocity of an object changes from to during an interval of t=t1 to t=tz, the
Average Acceleration is given as
𝑎⃑ Average=<𝑎⃑ >= =

𝑎⃑ Instantaneous = 𝑎⃑ = lim =

You know that the definitions might be boring but simple but applications are always interesting.
Therefore, let’s come to application
For Motion in One Dimension you have already learnt Equations of Motion in One Dimension
v=u+at s = ut + at2 vz = uz + 2as
We can not only establish same set of equation without graphs but we can derive any result if we use
Calculus Technique.
Calculus Techniques:– The Equations of Motion in One Dimension are consistent to constant
acceleration only. Therefore to start with consider constant acceleration.
i.e. 𝑎⃑ = constant t Therefore by definition For 2nd Equation we can use 𝑎⃑ =

∴ d𝑣⃑ =𝑎⃑dt⟹ ∫ 𝑑𝑣⃑ = ∫ 𝑎⃑. 𝑑𝑡 ⟹[𝑣⃑] =𝑎⃑[𝑡]


⟹ 𝑣⃑ − 𝑢⃑ = 𝑎⃑ = 𝑎⃑(𝑡 − 0) ⟹ 𝑣⃑ = 𝑢⃑ + 𝑎𝑡

70
Therefore you got your 1st Equation 𝑣⃑ = 𝑢⃑ + 𝑎⃑t or v = u + at because for Motion in One Dimension only
“+ve” & “–ve” sign are more than sufficient to rep represent direction. “+ve” means away from Origin
i.e. towards Right or Upwards and “–ve” means towards the Origin i.e. towards left or Downwards. 𝑣⃑ =
𝑢⃑ + 𝑎⃑t Sufficient for any type of motion with Constant Acceleration.

For 2nd Equation use 𝑣⃑ =
𝑡 𝑟⃑[𝑟 𝑡 𝑟⃑
∴ d𝑟⃑ = 𝑣⃑𝑑𝑡 ⟹ ∫0 𝑣⃑ ⟹ ∫𝑟⃑ 𝑑𝑟⃑ = ∫0 𝑢⃑ + 𝑎⃑𝑡 . 𝑑𝑡 ⟹ 𝑟⃑ 𝑟⃑𝑜
= 𝑢⃑. 𝑡 + 𝑎⃑. 𝑡
0

⟹ 𝑟⃑ − 𝑟⃑ = 𝑢⃑. 𝑡 + 𝑎⃑. 𝑡 Where 𝑟⃑ − 𝑟⃑ =𝑠⃑ (Displacement)


Therefore you got your 1st Equation 𝑣⃑ =𝑢⃑ + 𝑎⃑𝑡 = v = u + at because for Motion in One Dimension only
“+ve”&”–ve “sign are more than sufficient to represent direction. ”+ve” means away from Origin i.e.
towards Right or Upwards and “–ve” means towards the Origin i.e. towards left or Downwards.

NOTE:𝑣⃑ =𝑢⃑ + 𝑎⃑𝑡 is sufficient for any type of motion with Constant Acceleration.

For 2nd Equation use 𝑣⃑ =
⃑ ⃑
∴ d𝑟⃑=𝑣⃑. 𝑑𝑡 ⟹ ∫⃑ 𝑑𝑟⃑ = ∫ 𝑣⃑ 𝑑𝑡 ⟹ 𝑑𝑟⃑ = ∫ (𝑢⃑ + 𝑎⃑𝑡)𝑑𝑡 ⟹ [𝑟⃑] ⃑

= 𝑢⃑. 𝑡 + 𝑎⃑. 𝑡

⟹ 𝑟⃑ − 𝑟⃑ = 𝑢⃑. 𝑡 + 𝑎⃑. 𝑡 Where 𝑟⃑ − 𝑟⃑ = 𝑠⃑ (Displacement)


Therefore you got your 2nd Equation.𝑠⃑ = 𝑢⃑. 𝑡 + 𝑎⃑. 𝑡 or s = u.t. + a.t2 because for Motion in One
Dimension only “+ve”&”–ve” sign are more than sufficient to represent direction. “+ve” means away
from Origin i.e. towards Right or Upwards and “– ve” means towards the Origin i.e. towards left or
Downwards.
Note:𝑠⃑ = 𝑢⃑. 𝑡 + 𝑎⃑. 𝑡 is sufficient for any type of motion with Constant Accelection.

For 3rd Equation method is little bit different. As we know by definition 𝑎⃑ = but we require
Velocity and Displacement form.
⃑ ⃑ ⃑
Therefore 𝑎⃑ = × ⟹ 𝑎⃑ = 𝑣⃑ . 𝑎⃑. 𝑑𝑟⃑ ⟹ 𝑣⃑.d𝑣⃑

⃑ ⃑ ⃑
=∫ 𝑣⃑𝑑𝑣⃑ = 𝑎⃑ ∫⃑ 𝑑𝑟⃑⟹ = 𝑎⃑. [𝑟⃑] ⃑⃑

⟹ 𝑣⃑ 𝑢⃑ =2.𝑎⃑(𝑟⃑ − 𝑟⃑ ) Where 𝑟⃑ − 𝑟⃑ =𝑠⃑ (Displacement)

71
Therefore you got your 3rd Equation. 𝑣⃑ = 𝑢 + 2. 𝑎⃑. 𝑠⃑or𝑣⃑ = 𝑢 + 2. 𝑎. 𝑠 because for Motion in
One Dimension only “+ve”&“–ve” sign are more than sufficient to represent direction. “+ve” means
away from Origin i.e. towards Right or Upwards and “– ve” means towards the Origin i.e. towards left
or Downwards.

NOTE: 𝑣⃑ = 𝑢 + 2. 𝑎⃑. 𝑠⃑ is sufficient for any type of motion with Constant Acceleration.
Spontaneous Acceleration: – Instantaneous Acceleration is the Acceleration of the object at a particular
instant. Here we get to use Calculus once more.
Sn = u + (2n – 1); Sn is displacement of the particle in the n–th second
The term n–th second is to be understood with clarity. This instant with duration of 1 second.
The instant with duration of ‘t’ seconds can have duration less than or greater than or equal to one
second. But the n–th second always has the interval “one second”.
The last equation should not be used to calculate displacement if the interval of time not equal to 1
second.

Example:

(i) A car travels for six seconds. The last second of travel is 6th second. (Interval of last second is
one second)
(ii) A vertical travel for 6.1 seconds. The last second of travel has an interval of one second. i.e. from
5.1 s to 6.1 s
(iii) A cyclist travels for 0.95 seconds. There is no last second, because the travel time is less than
one second.
To calculate displacement from t1 to t2 seconds, use

s = ut + at2 i.e. s1 = ut1 + a𝑡 and s2 = ut2+ a 𝑡 ⟹(s2–s1)

= u (t2–t1) + a (𝑡 –𝑡 )

To calculate displacement from 5.1s to 6.1s, also use s = ut + at2 along with

Sn = u + (2n –1)

To calculate displacement from 5s to 6s. Use Sn = u + (2n –1) with n= 6

For a body moving with uniform acceleration, average velocity is given by and by

definition, average velocity = ⟹ =

72
CLASS EXERCISE SOLVED :

Problem1. The displacement of a particle, moving in a straight line, is given by s = 2t2+2t+4


where is in metres and in seconds. The acceleration of the particle is
a) 2 m/s2 b) 4 m/s2 c) 6 m/s2 d)8 m/s2
Solution: (b) Given s = 2t2+2t+4 velocity (v) = = 4𝑡 + 2and acceleration
(a) = 4(1) + 0 =4 m/s2

Problem 2 .The position x of a particle varies with time t as x= at2–bt3. The acceleration of the
particle will be zero at time t equal to
a) b) c) d) Zero
3𝑏
Solution: (c) Given x = at2–bt3 velocity (v) = = 2𝑎𝑡 − 3𝑏𝑡 and acceleration
(a) = 2𝑎 − 6𝑏𝑡
When acceleration = 0  2a–6bt  t= =
6𝑏

Problem3.The displacement of the particle is given by y = a+bt+ct2–dt4 .The initial velocity and
acceleration are respectively
a) b, –d b)–b, 2c c) b, 2c d) 2c, – 4d
Solution: (c) Given y = a+bt+ct2–dt4∴ v= = 0+b+2ct –4dt3
Putting vinitial = b
So initial velocity = b
𝑑𝑣
Now, acceleration (a) = = 0 + 2𝑐 − 12𝑑𝑡
Putting t = 0, initial = 2c

Problem4. The relation between time t and distance x is t =𝛼𝑥 + 𝛽𝑥, where 𝛼 and 𝛽are constants. The
retardation is (v is the velocity)
a) 2𝛼𝑣 b) 2𝛽𝑣 c)2𝛼𝛽𝑣 d) 2 𝛽 𝑣
Solution: (a) differentiating time with respect to distance = 2𝛼𝑥 + 𝛽
⟹𝑣 = =
.
So, acceleration (a) = = . =𝑣 =( = −2𝛼. 𝑣. 𝑣 = −2𝛼𝑣
𝑑𝑡 )

73
Problem 5.If displacement of a particle is directly proportional to the square of time. Then particle is
moving with
a) Uniform acceleration b) Variable acceleration
c) Uniform velocity d) Variable acceleration but uniform velocity
Solution: (a) Given that x t2 or x = kt2 (where K= constant) Velocity (v) = = 2𝐾𝑡 and Acceleration
(a) = = 2𝑘It is clear that velocity is time dependent and acceleration does not depend on time.
So we can say that particle is moving with uniform acceleration but variable velocity.

Problem 6. A particle is moving eastwards with velocity of 5 m/s. In 10 sec the velocity changes to 5 m/s
northwards. The average acceleration in this time is
a) Zero b) m/s2 toward north–west

(c) m/s2toward north–east d) m/s2 toward north–west



Solution: (b)Δ𝜐̅ = 𝜐̅ 2–𝜐̅ 1

Δ𝜐 = 𝜐 + 𝜐 − 2𝜐 𝜐 𝑐𝑜𝑠90 = 5 + 5 = 5√2


v1

Δ𝜐 = 5√2 
 2  5m / s

90o

1  5 m / s


Average acceleration= = = m/s2 toward north–west (As clear from the figure).

Problem7. A body starts from the origin and moves along the x–axis such that velocity at any instant is
given by, where t is in second and velocity is in m/s. What is the acceleration of the particle, when it is
2m from the origin?
a) 28 m/s2 b) 22 m/s2 c)12 m/s2 d) 10 m/s2
Solution: (b) Given that 𝜐 = 4𝑡 − 2𝑡
at∫ 𝜐 𝑑𝑡𝑑𝑥 𝑥 = 𝑡 − 𝑡 + 𝑐 at t=0, x=0 ⟹ 𝐶 = 0
When particle is 2m away from the origin 2 = t4–t2⟹ t4–t2 –2 =0 ⟹ (t2–2)(t2+1) = 0⟹t = √2sec
a= = (4𝑡 − 2𝑡) = 12𝑡 − 2 ⟹ 𝑎 = 12𝑡 − 2
For t = √2sec ⟹a =12× (√2)2–2⟹a = 22 m/s2

74
Problem8.A body of mass 10 kg is moving with a constant velocity of 10 m/s. When a constant force acts
for 4 sec on it, it moves with a velocity 2 m/sec in the opposite direction. The acceleration produced in
it is
a) 3 m/s2 b) –3m/s2 c) 0.3m/s2 d)–0.3 m/s2
Solution: (b) Let particle moves towards east and by the application of constant force it moves towards
west
𝜐̅ 1 =+10 m/s and 𝜐̅ 2= –2 m/s. Acceleration= =
CLASS EXERCISE
1] Moving with uniform acceleration, a body covers 150m during 10sec so that it covers 24 m during the
tenth second. Find the initial velocity and acceleration of the body
a) 2 ms–1; 5 ms–2 b) 5ms–1; 2 ms–2 c) 3 ms–1; 4 ms–2 d) 4ms–1;3ms–2
2] A body moving with uniform acceleration covers 100m in the first 10 seconds and 150m in the next 10
seconds. The initial velocity of the body is
a) 15 ms–1 b) 7.5 ms–1 c) 5 ms–1 d) 2.5 ms–1
th
3] A particle moving with a constant acceleration describes in the last second of its motion of the
whole distance. If it starts from rest, how long is the particle in motion
a) 5s b) 10s c) 15s d) none
4] A body starts with initial velocity u and moves with uniform acceleration f. If when the velocity has
increased to 5u, the acceleration is reversed in direction, the magnitude remaining constant; it turns to
the starting point with velocity of
a) –u b) –6u c) –7u d) –9u
5] A body moving with uniform acceleration travels a distance Sn = (0.4n + 9.8) m in nth sec. Find the

initial velocity of the body in ms–1


a) 0.4 b) 10 c) 5 d) 4
6] The displacement of particle is zero at t = 0 and it is x at t = t. It starts moving in the positive x direction
with a velocity which varies as V = K, where K is constant. The relation between V and
t is
a) Kt/2 v) Kt3/2 c) K2t/2 d) 2Kt2
7] A particle starts from rest with constant acceleration for 20sec. If it travels a distance y1 in the first 10
sec and a distance y2 in the next 10 sec then
a) y2 =2y1 b) y2 =3y1 c) y2 =4y1 d) y2 =5y1

75
8] The ends of a moving train with a constant acceleration pass a certain point with velocities u and v.
What is the velocity V with which the mid–point of the train passes through the same point?

a) v = b) v = √𝑢 + 𝑣 c) v = d) V =

9] The velocity of a body at an instant is 20 ms–1. After 5s the velocity is 30 ms–1. How many seconds
earlier from the instant, it might have started? Assume acceleration is uniform
a) 8sec b) 10sec c) 6sec d) 7sec
10] The engine of a train moving with uniform acceleration passes an electric pole with a
velocity u and the last compartment with a velocity V, the length of train that has passed the
pole when the velocity is , if the total length of the train is l
( ) ( )
a) b) c) d)
( ) ( )

HOME EXERCISE
1] The motion of a particle along X–axis is given by the equation X = 9 + 5t2, where X is distance in cm
and t is time in sec. Average velocity during the interval from t = 3 second t = 5 second is
a) 40 cms–1 b) 30 cms–1 c) 35 cms–1 d) 50 cms–1
2] A bus accelerates uniformly from rest and acquires a speed of 72 km/hr in 20 sec. The acceleration of
the bus is
a) 10 m/s2 b) 5 m/s2 c) 2m/s2 d) 1 m/s2
3] Speeds of two identical cars are U and 4U at a specific instant. The ratio of the respective distances in
which the two cars are stopped from that instant is
a) 1 : 1 b) 1 : 4 c) 1 : 8 d) 1 : 16
4] A car moving with a speed of 50 kmph can be stopped by breaks after at least 6m. If the same car is
moving at a speed of 100kmph, the minimum stopping distance is
a) 12 m b) 18m c) 24m d) 6 m
5] If a body loses half of its velocity on penetrating 3 cm in a wooden block, then how much will it
penetrate more before coming to rest?
a) 1 cm b) 2cm c) 3 cm d) 4 cm
6] The distance travelled by a body is directly proportional to the square of the time taken. Its acceleration
a) increases b) decreases c) becomes zero d) remains constant
7] A particle moves along a straight line such that its displacement ‘x’ varies with time‘t’ as x = a + bt+
ct2. Then its acceleration at t = 2s and average velocity during 3rd second are
(a, b,c are constants)
a) 2c, (b + 5c) b) c, (b+c) c) (b +5c), 2c d) 2c, (5b + c)

76
8] A body moving with a uniform acceleration covers 6m and 10m during successive seconds. Its
acceleration is
a) 1 m/s2 b) 16 m/s2 c) 4 m/s2 d) 6 m/s2
9] A body travels 2 m in the first two seconds and 2.20 m in the next 4 seconds with uniform deceleration.
The velocity of the body at the end of 9 seconds is
a) –10 ms–1 b) –0.20 ms–1 c) –0.40 ms–1 d) –0.80 ms–1
10] A particle moves 10m in the first 2 second, 20m in the next 3 second and 30m in the next 10 second.
Which of the following statements are true for this motion?
a) The particle was uniformly accelerated
b) The particle had decelerating forces acting on it during the motion
c) The average speed of the particle was 4 m/s
d) The average acceleration of the particle was 3 m/s2
11] A bee flies in a line from a point A to another point B in 4s with a velocity of |t – 2| m/s. The distance
between A and B in metre is
a) 2 b) 4 c) 6 d) 8
12] A bullet loses1/40 of its initial velocity while passing through a plank. The least number of planks
required to just stop the motion of bullet is
a) 20 b) 21 c) 10 d) 11
13] A train accelerates from rest at a constant rate 𝛼 for some time and then it retards to rest at the constant
rate 𝛽. If the total distance covered by the particle is x, then what is the maximum velocity of the train?
/ 2𝛼𝛽 / / /
a) 𝑥 b) 𝑥 c) 𝑥 d) 𝑥
2𝛼𝛽
14] A particle starts from rest and travels “S” distance with uniform acceleration and “5S” distance with
maximum velocity and then with uniform deceleration a distance “3S” and comes to rest. Find the ratio
of average velocity to comes to rest: Find the ratio of average velocity to maximum velocity
a) 2/5 b) 7/13 c) 2/13 d) 9/13
15] Two motor cars start off with a gap 2 min. with the same acceleration. How long after the departure of
th
the second car does travel a distance equal to 1/9 the distance covered by the first car
a) 1/2 min b) 1 min c) 1 ½ min d) 2 min
st
16] Two bodies move in the same straight line at the same instant of time from the same origin. The 1
nd
body moves with a constant velocity of 40 m/s and the 2 starts with a constant acceleration 4 m/s2.
Find the time at which the two bodies meet
a) 5 sec b) 10 sec c) 20 sec d) 30 sec

77
17] A truck starts from rest with an acceleration of 1.5 m/s2 while a car 150m behind starts from rest with
an acceleration of 2 m/s2 at the same instant of time. How much distance is covered by the truck before
they are side by side?
a) 150m b) 300m c) 450m d) 600m
18] Two cars X and Y start off to a race on a straight path with initial velocities of 8 m/s and 5 m/s
respectively. Car X moves with uniform acceleration of 1 m/s2 and car Y moves with uniform
acceleration 1.1 m/s2. If both the cars reach the winning post together, find the length of the track?
a) 1000m b) 2000m c) 2500m d) 2280m
19] Two cars traveling towards each other on a straight road at velocities of 10 m/s and 12 m/s respectively.
When they are 150m apart, both drivers apply their brakes and each car decelerates at 2 m/s2 until it
stops. How far apart will they be when they have both come to stop?
a) 60 m b) 72 m c) 80 m d) 89 m
20] The reaction time for an automobile driver is 0.7 sec. If the automobile can be decelerated at 5m/s2,
calculate the total distance traveled in coming to stop from an initial velocity of 10m/s after a signal is
observed.
a) 10 m b) 17 m c) 70 m d) 19.2 m

78
SESSION– 6
AIM
 Motion with Variable Acceleration

Motion with Variable Acceleration


(i) If acceleration is a function of time then a = f (t) then v = u +∫ 𝑓(𝑡)𝑑𝑡
and s = ut +∫(∫ 𝑓(𝑡) 𝑑𝑡) 𝑑𝑡
(ii) If acceleration is a function of distance a = f(x) then v2=u2+2∫ 𝑓(𝑥)𝑑𝑥

(iii) If acceleration is a function of velocity a = f (v) then t = ∫ and x = x0 + ∫


( ) ( )

CLASS EXERCISE:

Problem1. An electron starting from rest has a velocity that increases linearly with the time that is v = kt
where k = 2 m/sec2. The distance travelled in the first 3 seconds will be
a) 9 m b) 16 m c) 27 m d) 36 m

Solution: (a) x = ∫ 𝑣 𝑑𝑡 = ∫ 2𝑡 𝑑𝑡 = 2 = 9m

Problem2.The acceleration of a particle is increasing linearly with time t as bt. The particle starts from the
origin with an initial velocity The distance travelled by the particle in time t will be
a) V0t + bt2 b) V0t + bt3 c) V0t + bt3 d) V0t + bt2

Solution: (c) ∫ 𝑑𝑣=∫ 𝑎𝑑𝑡 = ∫ (𝑏𝑡)𝑑𝑡

⟹ v2 – v1= 𝑣 =𝑣 + =𝑣 +

S = ∫ 𝑣 𝑑𝑡 + ∫ 𝑑𝑡 = 𝑣 𝑡 + 𝑏𝑡

Problem3. The motion of a particle is described by the equation u = at. The distance travelled by the
particle in the first 4 seconds
a) 4𝛼 b)12 𝛼 c)6 𝛼 d) 8 𝛼
Solution: d) u = at⟹ =at

S= ∫ 𝑎𝑡 𝑑𝑡= a =8a

79
Problem4. A body A moves with a uniform acceleration and zero initial velocity. Another body B starts
from the same point moves in the same direction with a constant velocity v. The two bodies meet after a
time t. The value of t is

a) b) c) d)

Solution: (a) Let they meet after time’t’. Distance covered by body A 𝑎𝑡 Distance covered by body B=
vt and 𝑎𝑡 = 𝑣𝑡 ∴ 𝑡 =

Problem5. A student is standing at a distance of 50metres from the bus. As soon as the bus starts its motion
with an acceleration of 1ms-2, the student starts running towards the bus with a uniform velocity
Assuming the motion to be along a straight road, the minimum value of , so that the students is able to
catch the bus is
a) 5 ms-1 b)8 ms-1 c) 10 ms-1 d)12 ms-1
Solution: (c) Let student will catch the bus after sec. So it will cover distance ut.
Similarly distance travelled by the bus will be 𝑎𝑡 for the given condition

ut = 50 + 𝑎𝑡 =50 + = + (As a = 1 m/s2)

Problem6. A particle is moving eastwards with velocity of 5 m/s. In 10 sec the velocity changes to 5 m/s
northwards. The average acceleration in this time is
a) Zero b) toward north-west

c) m/s2toward north-east d) m/s2 toward north-west



Solution: b)Δ𝜐⃑ = 𝜐 ⃑ – 𝜐 ⃑

Δ𝜐 = 𝜐 + 𝜐 − 2𝜐 𝜐 𝑐𝑜𝑠90 = 5 + 5 = 5√2

v1


 2  5m / s

90o

1  5 m / s


Δ𝜐 = 5√2 Average acceleration = = = 𝑚/𝑠 toward north-west (As clear from the figure).

80
Problem7. A body starts from the origin and moves along the x-axis such that velocity at any instant is
given by where t is in second and velocity is in m/s. What is the acceleration of the particle, when it is
2m from the origin?
a) 28 m/s2 b) 22 m/s2 c) 12 m/s2 d) 10 m/s2
Solution: (b) Given that 𝜐 = 4𝑡 − 2𝑡
x = ∫ 𝜐 𝑑𝑡𝑑𝑥at x = t4-t2+c at t =0, x=0 ⟹C=0
When particle is 2m away from the origin
2= t4–t2 ⟹t4–t2–2=0 ⟹ (t2–2) (t2+1) = C ⟹ t=√2 𝑠𝑒𝑐sec
a= = (4𝑡 − 2𝑡) = 12𝑡 − 2 ⟹ 𝑎 = 12𝑡 − 2for
t = √2 sec ⟹a = 12×(√2)2–2⟹ 𝑎=22 m/s2

Problem 8. A body of mass 10 kg is moving with a constant velocity of 10 m/s. When a constant force acts
for 4 sec on it, it moves with a velocity 2 m/sec in the opposite direction. The acceleration produced in
it is
a) 3 m/s2 b) –3 m/s2 c) 0.3 m/s2 d) –0.3 m/s2
Solution: (b) Let particle moves towards east and by the application of constant force it moves towards
west
⃑ ⃑
𝜐 ⃑ = +10 𝑚/𝑠dxand 𝜐 ⃑ = −2𝑚/𝑠Acceleration = =
𝑇𝑖𝑚𝑒
(−2) − (10) 12
⟹𝑎= =− = −3 𝑚/𝑠
4 4

Problem9. A car, moving with a speed of 50 km/hr, can be stopped by brakes after at least 6m. If the same
car is moving at a speed of 100 km/hr, the minimum stopping distance is
a) 6m b) 12m c) 18m d) 24m

Solution (d): v2 = u2 - 2as ⟹ s = ⟹s  u2 (As a = constant)

= ⟹ s2 = 4s1 = 4 x 12 = 24m

Problem10. The velocity of a bullet is reduced from 200m/s to 100m/s while travelling through a wooden
block of thickness 10cm. The retardation, assuming it to be uniform, will be
a) 10 x 104 m/s2 b) 12 x 104 m/s2 c) 13.5 x 104 m/s2 d) 15 x 104 m/s2
( ) ( )
Solution: (d) u = 200 m/s, v = 100 m/s, s = 0.1 m; a = = = 15 x 104 m/s2
× .

81
Problem 11.A body A starts from rest with an acceleration. After 2 seconds, another body B starts from rest
with an acceleration. If they travel equal distances in the 5th second, after the start of A, then the ratio
is equal to
a) 5 : 9 b) 5 : 7 c) 9 : 5 d)9 : 7
Solution: (a) By using Sn = u+ (2n -1), Distance travelled by body A in 5th second= 0 + (2 x 5 -1)

Distance travelled by body B in 3rd second is = 0+ (2×3–1)


According to problem: 0+ (2×5–1) = 0+ (2×3–1)⟹ 9 a1 = 5a2 ⟹ =

Problem12. The average velocity of a body moving with uniform acceleration travelling a distance of 3.06
m is 0.34 ms-1. If the change in velocity of the body is 0.18ms-1 during this time, its uniform
acceleration is
a) 0.01 ms-2 b) 0.02 ms-2 c) 0.03 ms-2 d) 0.04 ms-2
.06
Solution: (b) Time = = =9sec
𝐴𝑣𝑒𝑟𝑎𝑔𝑒 .

.
and Acceleration = = =0.02 m/s2.
𝑇𝑖𝑚𝑒

Problem13. A particle travels 10m in first 5 sec and 10m in next 3 sec. Assuming constant acceleration
what is the distance travelled in next 2 sec
a) 8.3 m b) 9.3 m c) 10.3 m d) None of above
Solution: a) Let initial (t = 0) velocity of particle = u for first 5sec of motion s5 = 10 metre, so by using
s= ut + at2
10 = 5u + a (5)2⟹ 2u + 5a = 4 ......... (i)
for first 8 sec of motion s8 = 20 metre

20 = 8u + a (8)2⟹ 2u + 8a = 5 ........ (ii)

By solving (i) and (ii) u = m/s a = m/s2

Now distance travelled by particle in total 10 sec. s10 = u×10+ a(10)2


by substituting the value of u and a we will get s10 = 28.3 m
So the distance in last 2 sec = s10 - s8 = 28.3 - 20 = 8.3 m

82
Problem14. A body travels for 15 sec starting from rest with constant acceleration. If it travels distances S1,
S2 and S3 in the first five seconds, second five seconds and next five seconds respectively the relation
between S1, S2 and S3 is

a) S1 = S2 = S3 b) 5S1 = 3S2 = S3 c) S1 = S2 = d) S1 = S2 = S3
Solution: (c)Since the body starts from rest. Therefore u = 0

25𝑎
S1 = a(5)2 =

100𝑎 100𝑎
S1 + S2 = a (10)2 = ⟹ S2 = -S1 = 75
225𝑎 225𝑎 125𝑎
S1 + S2 + S3 = a (15)2 = ⟹S3 = -S2 - S1 =

Thus Clearly S1 = S2 = S3

Problem15. If a body having initial velocity zero is moving with uniform acceleration the distance travelled
by it in fifth second will be
a) 36 metres b) 40 metres c) 100 metres d) Zero
Solution: (a)sn= u + a (2n - 1) = 0 + (8) [2 x 5 - 1] = 36 metres

Problem16. The engine of car produces acceleration 4m/sec2 in the car, if this car pulls another car of same
mass, what will be the acceleration produced
a) 8 m/s2 b) 2 m/s2 c) 4 m/s2 d) m/s2

Solution: (b) F = ma a if F = constant. Since the force is same and the effective mass of system

becomes double = = a2= =2m/s2

Problem17. A body starts from rest. What is the ratio of the distance travelled by the body during the 4th
and 3rd second?
a) 7/5 b) 5/7 c)7/3 d)3/7
Solution: (a) As Sn(2𝑛 − 1), =

83
CLASS EXERCISE
1] If the relation between distance x and time t is of the from t = 𝛼x2+𝛽x here 𝛼 and 𝛽 being appropriate
constants, then the retardation of the particle is
3 3 2 3
a) 2𝛼 v3 b) 2 𝛽 v c) 2𝛼𝛽v d)2𝛽 v
2] A particle moves along a straight line according to the law 𝑆 = 𝑎𝑡 + 2𝑏𝑡 + 𝑐. . The acceleration of the
particle varies as
a) S–3 b) S2/3 c) S3 d) S5/2
3] A point moves rectilinearly with deceleration whose modulus depends on the velocity v of the particle
as 𝛼 = 𝑘 √𝑣, where k is a positive constant. At the initial moment the velocity of the point is equal to
v0. What distance will it traverse before it stops? What time will it take to cover that distance?

4] If the displacement of the particle varies with time as x1/2=t+7, then


a) velocity of the particle is inversely proportional to t
b) velocity of the particle is proportional to t
c) velocity of the particle is inversely proportional t1/2.
d) the particle moves with a constant acceleration.
5] A point moves in a straight line under the retardation kv2. If the initial velocity is u, the distance
covered in t second is

a) kut b) log (k ut) c) log(1+ kut) d) k log (k ut)

6] A particle of mass 10–2 kg is moving along the positive x-axis under the influence of a force
F(x) = – (k/2x2) where k = 10–2 Nm2. At time t = 0 it is at x = 1 m and its velocity v = 0.
(i) Find its velocity when it reaches x = 0.5 m
7] A particle moving in a straight line has an acceleration of (3t - 4) ms-2 at timet seconds. The particle is
initially 1m from O, a fixed point on the line, with a velocity of 2 ms-1. Find the times when the
velocity is zero. Find also the displacement of the particle from O whent = 3.

84
HOME EXERCISE
1] A point moves such that its displacement as a function of time is given by x2 = t2 + 1. Its
acceleration at time t is.

a) b) __ c) − d) −
2] A particle moves in a straight line so that after t second, the distance x from a fixed point O on the line
is given by x = (t – 2)2 (t – 5). Then
a) after 2s, velocity of particle is zero b) after 2s, the particle reaches at O
c) the acceleration is negative, when t < 3 s d) all the above
3] The initial velocity of a particle of mass 2kg is (4𝚤̂ + 4𝚥̂) m/s. A constant force of –20𝚥̂N is applied on
the particle. Initially, the particle was at (0, 0). Find the x-coordinate of the point, where its y-coordinate
is again zero
a) 3.2 m b) 1.6 m c) 4.8 m d) 1.2 m

85
SESSION –7, 8AND9

AIM
 Graphical Representation of motion
Position – Time Graph:
During motion of the particle its parameters of kinematical analysis (u, v, a, r) changes with time. This
can be represented on the graph.
Position time graph is plotted by taking time t along x-axis and position of the particle on y-axis.
Let AB is a position-time graph for any moving particle
As Velocity =
P
= ----------- (i)

O
𝜃 = constant so v = constant, a = 0 i.e., line with constant slope
T
represents uniform velocity of the particle.
P

O 𝜃is increasing so v is increasing, a is positive. i.e., line bending


T
towards position axis represents increasing velocity of particle. It
P means the particle possesses acceleration.

O 𝜃 is decreasing so v is decreasing, a is negative i.e., line bending


T

towards time axis represents decreasing velocity of the particle. It


means the particle possesses retardation.
P
𝜃constant but > 90o so v will be constant but negative
 i.e., line with negative slope represent that particle returns towards
O T
the point of reference. (negative displacement).

A B
C
Straight line segments of different slopes represent that velocity of
O T
S the body changes after certain interval of time.
P

T
O
This graph shows that at one instant the particle has two positions.
P Which is not possible?

O
T The graph shows that particle coming towards origin initially and after
that it is moving away from origin.

86
Note: If the graph is plotted between distance and time then it is always an increasing curve and it never
comes back towards origin because distance never decreases with time. Hence such type of distance
time graph is valid up to point A only, after point A it is not valid as shown in the figure.
For two particles having displacement time graph with slopes 𝜃 1 and 𝜃2 possesses velocities v1 and v2

respectively then =
𝑡𝑎𝑛

CLASS EXERCISE SOLVED

Problem1. The position of a particle moving along the x-axis at certain times is given below:
t (s) 0 1 2 3
x (m) -2 0 6 16

A
Distance

O Time

a) Uniform, accelerated b) Uniform, decelerated


c) Non-uniform, accelerated d)There is not enough data for generalisation
Solution: (a) Instantaneous velocity 𝑣 = By using the data from the table
( )
𝑣 = = 2 𝑚/𝑠, 𝑣 = = 6 𝑚/𝑠and𝑣 = = 10 m/s i.e. the speed is increasing at
a constant rate so motion is uniformly accelerated.

Problem2. Which of the following graph represents uniform motion?

s s
s s

a) b) c) d)
t t t
t

Solution: (a) When distance time graph is a straight line with constant slope than motion is uniform.

87
Problem3. The displacement-time graph for two particles A and B are straight lines inclined at angles of
30o and 60o with the time axis. The ratio of velocities of vA: vB is
a) 1 : 2 b) 1:√3 c) √3 :1 d)1 : 3
/√
Solution: (d)𝑣 = 𝑡𝑎𝑛𝜃from displacement graph so = = = =
√ √ ×√

Problem4. From the following displacement time graph find out the velocity of a moving body
Time (sec)

30o
O
Displacement (meter)

a) m/s b) 3 m/s c) √3m/s d)


Solution: (c) In first instant you will apply 𝜐 = 𝑡𝑎𝑛𝜃 and say 𝜐 = 𝑡𝑎𝑛30 = m/s

But it is wrong because formula is valid when angle is measured with time axis.
Here angle is taken from displacement axis. So angle from time axis
Now

Problem5. The diagram shows the displacement-time graph for a particle moving in a straight line. The
average velocity for the interval t = 0, t = 5 is

x
20

10
5
O t
2 4
– 10

a) 0 b) 6 ms-1 c) - 2 ms-1 d) 2 ms-1

( ) ( ) (
Solution: (c) Average velocity = = -2 m/s
𝑇𝑜𝑡𝑎𝑙

88
Problem6. Figure shows the displacement time graph of a body. What is the ratio of the speed in the first
second and that in the next two seconds?

Y
30

Displacement
20

10

0 X
1 2 3
Time

a) 1 : 2 b) 1 : 3 c) 3 : 1 d) 2 : 1
Solution: (d) Speed in first second = 30 and Speed in next two seconds = 15. So that ratio 2: 1

Velocity -Time Graph.


The graph is plotted by taking time t along x-axis and velocity of the particle on y-axis. Distance and
displacement: The area covered between the velocity time graph and time axis gives the displacement
anddistance travelled by the body for a given time interval.

Then Total distance =|𝐴 | + |𝐴 | + |𝐴 | = Addition of modulus of different area. i.e


s =∫|𝜐| 𝑑𝑡

+

1 3
t
2

–

Total displacement =𝐴 + 𝐴 + 𝐴
= Addition of different area considering their sign. i.e. r =∫ 𝜐 𝑑𝑡
Here A1 and A2 are area of triangle 1 and 2 respectively and A3 is the area of trapezium .Acceleration:
Let AB is a velocity-time graph for any moving particle
As Acceleration = = … (i)
𝑇𝑎𝑘𝑒𝑛

89
y

v2 D

Velocity
B

v1 
C
A

O x
t1 t2
Time

From triangle ABC, tan𝜃 = = =


By comparing (i) and (ii)
Acceleration (a) = tan𝜃

It is clear that slope of velocity-time graph represents the acceleration of the particle.
Velocity

𝜃 = 0, a = 0, v = constant i.e., line parallel to time axis


O
represents that the particle is moving with constant
velocity.
Velocity

O 𝜃 = 90o, a = , v = increasing i.e., line perpendicular to time axis


Time
represents that the particle is increasing its velocity, but time does
not change. It means the particle possesses infinite acceleration.
Practically it is not possible.
Velocity

O 𝜃 = constant, so a = constant and v is increasing uniformly with time i.e.,


Time
line with constant slope represents uniform acceleration of the particle.
Velocity

𝜃increasing so acceleration increasing i.e., line bending towards velocity


O
axis represent the increasing acceleration in the body.
Time

𝜃decreasing so acceleration decreasing i.e. line bending towards time axis


Velocity

represents the decreasing acceleration in the body


O Time

90
Velocity

O
Time
Positive constant acceleration because is constant and < 90o but initial
Velocity of the particle is negative.
Velocity

O
Positive constant acceleration because is constant and < 90o but initial
Time

velocity of particle is positive.


Velocity

Negative constant acceleration because is constant and > 90o but initial
O
Time velocity of the particle is positive.
V elocity

Negative constant acceleration because is constant and > 90o but initial
O
Time velocity of the particle is zero.
Velocity

Negative constant acceleration because 𝜃 is constant and > 90o but initial
O
Time velocity of the particle is negative.

CLASS EXERCISE SOLVED

Problem 1.A ball is thrown vertically upwards. Which of the following plots represents the speed-time
graph of the ball during its flight if the air resistance is not ignored?
a) b) c) d)
Speed

Speed
Speed

Speed

Time Time
Time Time

Solution: (c) In first half of motion the acceleration is uniform & velocity gradually decreases, so slope will
be negative but for next half acceleration is positive. So slope will be positive. Thus graph 'C' is correct.

91
Not ignoring air resistance means upward motion will have acceleration (a + g) and the downward
motion will have
Problem2. A train moves from one station to another in 2 hour’s time. Its speed-time graph during this
motion is shown in the figure. The maximum acceleration during the journey is

100
Speed in km/hours

80

60 D

40
20 B C L

N M E
A
0.25 0.75 1.00 1.5 2.00
Time in hours

a) 140 km h-2 b) 160 km h-2 c) 100 km h-2 d)120 km h-2

Solution: (b) Maximum acceleration means maximum slope in speed - time graph. That slope is for line
CD. amax=
Slope of CD = = 160 𝑘𝑚ℎ
. . .

Problem3. The graph of displacement v/s time is

Its corresponding velocity-time graph will be

a) V b) c) d)
V V V

t
t t t

92
Solution: (a) We know that the velocity of body is given by the slope of displacement - time graph. So it is
clear that initially slope of the graph is positive and after some time it becomes zero (corresponding to
the peak of the graph) and then it will be negative.
Problem4. In the following graph, distance travelled by the body in metres is

Y
15
v (m /s )

10
5
X
0 10 20 30
Time (s)

a) 200 b) 250 c) 300 d) 400


Solution: (a) Distance = The area under v - t graph
S = (30+10)×10= 200 metre

Problem5. For the velocity-time graph shown in figure below the distance covered by the body in last two
seconds of its motion is what fraction of the total distance covered by it in all the seven seconds

B C
10
v (m/s )

5
A P Q D
0 1 3 5 7
Time (s)

a) b) c) d)
Solution: (b) Distance covered in total 7 seconds = Area of trapezium ABCD
= (2 + 6) × 10 40m

Distance covered in last 2 second = area of triangle CDQ × 10 = 10 m


10
So required fraction = =

93
Problem6. The velocity time graph of a body moving in a straight line is shown in the figure. The
displacement and distance travelled by the body in 6 sec are respectively

A B
4
G H
2
v (m/s)

2 F 6
O C 4 I

–2 D E t (sec)
–4

a) 8m , 16 m b) 16 m , 8 m c)16m, 6m d) 8m, 8m
Solution: (a) Area of rectangle ABCO = 4 × 2 = 8 m
Area of rectangle CDEF = 2 × (- 2) = - 4 m
Area of rectangle FGHI = 2×2 = 4 m
Displacement = sum of area with their sign = 8 + (- 4) + 4 = 8 m
Distance = sum of area without sign = 8 + 4 + 4 = 16 m

Problem7. A ball is thrown vertically upward which of the following graph represents velocity time graph
of the ball during its flight (air resistance is neglected)

a) b) c) d)
V elocity

V e lo c ity
V e lo city

Velocity

Time Time Time Time

Solution: (d) In the positive region the velocity decreases linearly (during rise) and in negative region
velocity increase linearly (during fall) and the direction is opposite to each other during rise and fall,
hence fall is shown in the negative region.

94
Problem8. A ball is dropped vertically from a height d above the ground. It hits the ground and bounces up

vertically to a height . Neglecting subsequent motion and air resistance, its velocity v varies with the
height h above the ground as.

 
 

a) d b) c) d)d/2
h h d/2 d
d/2 h d h
d/2 d

Solution: (a) When ball is dropped from height d its velocity will be zero.
As ball come downward h decreases and increases just before the rebound from the earth
h = 0 and v = maximum and just after rebound velocity reduces to half and direction becomes opposite.
As soon as the height increases its velocity decreases and becomes zero at h =
This interpretation is clearly shown by graph (a).

Problem9. The acceleration-time graph of a body is shown below -

The most probable velocity-time graph of the body is

   

t t t t

(a) (b) (c) (d)

95
Solution: (c) From given a - t graph acceleration is increasing at constant rate
= 𝑘 (Constant) ⟹ a = kt (by integration)

⟹ = 𝑘𝑡⟹ dv =ktdt ⟹∫ 𝑑𝑣 = 𝑘 ∫ 𝑡 𝑑𝑡 ⟹ v =
i.e., v is dependent on time parabolically and parabola is symmetric about v-axis. and suddenly
acceleration becomes zero. i.e. velocity becomes constant. Hence (c) is most probable graph.

Problem10. Which of the following velocity time graphs is not possible?

v v v v

O t O t O t O t

(a) (b) (c) (d)

Solution: (d) Particle cannot possess two velocities at a single instant so graph (d) is not possible.

Problem11. For a certain body, the velocity-time graph is shown in the figure. The ratio of applied forces
for intervals AB and BC is


D

60o
30o
t
A B C

a) + b)− c) + d)−
Solution: (d) Ratio of applied force = Ratio of acceleration
𝑡𝑎𝑛30 /√
= = = = -1/3
( ) √

96
Problem12. Velocity-time graphs of two cars which start from rest at the same time are shown in the figure.
Graph shows, that

Velocity
A B

A

B

O t Time

a) Initial velocity of A is greater than the initial velocity of B


b) Acceleration in A is increasing at lesser rate than in B
c) Acceleration in A is greater than in B
d) Acceleration in B is greater than in A
Solution: (c) At a certain instant t slope of A is greater than B (𝜃 A,𝜃 B), so acceleration in A is greater than B

Problem13. Which one of the following graphs represents the velocity of a steel ball which falls from a
height on to a marble floor? (Here v represents the velocity of the particle and t the time)

Velocity

a) b) c) d)
Velocity
Velocity

Velocity

Time Time
Time Time

Solution: (a) Initially when ball falls from a height its velocity is zero and goes on increasing when it comes
down. Just after rebound from the earth its velocity decreases in magnitude and its direction gets
reversed. This process is repeated until ball comes to at rest. This interpretation is well explained in
graph (a).
Problem14. The adjoining curve represents the velocity-time graph of a particle, its acceleration values
along OA, AB and BC in metre/sec2 are respectively

A B
10
Velocity (m/sec)

O 10 20 30 40
Time (sec)

a) 1, 0, - 0.5 b) 1, 0, 0.5 c) 1, 1, 0.5 d) 1, 0.5, 0

97
Solution: (a) Acceleration along OA = = = 1 m/s2

Acceleration along OB = = 0; = -0.5 m/s2


Acceleration along BC=
20
Acceleration-time graph: It is a graph plotted between time and acceleration. If the graph is a line parallel
to time axis, the acceleration is constant. If it is a straight line with positive slope, the acceleration is
uniformly increasing. The co-relation of the graph explained above follows directly from the differential
𝑑𝑣
expressions v = and a = .
The area enclosed between acceleration-time graph and time axis gives during this time interval change
in velocity.

CLASS EXERCISE
1] The variation of velocity with time of a particle moving along a straight line is illustrated in the
following figure. The distance traveled by the particle in 4sec. is

30
A B
m/s

20 C
D
10
E
1 2 3 4
a) 60m b) 45m c) 55m d) 50m
2] In the given v-t graph, the distance travelled by the body in 5 second will be

a) 20 m b) 40 m c) 80 m d) 100 m
3] For the displacement- time graph shown in fig. the ratio of the magnitudes of the speeds during the first
two second and the next four second is

20m
displacement

0s 2s 6s
Time

a) 1: 1 b) 2 : 1 c) 1 : 2 d) 3 : 2

98
4] The velocity- time graph of a particle moving along a straight line is shown in fig. The displacement of
the body in 5 second is

2
1.5 1

V (m /s -1 )
0 4 5
1 2 3
1 t(ins)
1.5
2

a) 2.5m b) 1m c) 2m d) 3m

HOME EXERCISE
1] The velocity - time graph of a body is shown in figure. The displacement of the body in 8 seconds is:

6
(m/sec)

4
2
5 7
1 2 3 4 t
6 8 (Sec.)

-6
a) 10 m b) 9m c) 24m d) 30m
2] The v-t graph of a moving object is given in figure. The maximum acceleration is:

80
Velocity(m/S )
-1

60
40
20

10 20 30 40 50 60 70
Time (in Sec.)

a) 1 ms2 b) 2 ms2 c) 6 ms2 d)4ms2


3] Which of the following distance time graphs represents one dimensional uniform motion?

a) x b) c) x d)
x x

t t O t O t

99
4] Which of the following velocity time graphs is NOT possible

v v
v v
a) b) c) d)
t t
t t

5] Which of the following cannot be the speed time graph?

a) v b) v c) d) v
v

O t O t O t
O t

6] Which of the graphs below correctly shows how the acceleration and velocity of a perfectly elastic ball
bouncing on a horizontal surface varies with time?

t
t t

I II III
a) b) c) d)
Acceleration II III III I
Velocity I I II II

7] The speed versus time graph of a body is shown in fig. Which of the following statements is correct?

15
Speed ( in ms )
4

10

1 2 3 4
time(in s)

100
a) The body is moving with uniform acceleration of 6.67 ms-2 at all the times
b) The body is at rest for 1s and has a uniform acceleration of 6.67 ms-2.
c) The body is at rest for one second and has a uniform acceleration of 5 ms-2 afterward.
d) The body is at rest for 1 s and has a uniform retardation of 6.67 ms-2 afterwards

8] The v-t graph of a particle in linear motion is shown in adjoining figure. The distance from origin after
8 seconds is:
4
(m/s)

5 6 7 8 t
1 2 3 4 (sec)
2

a) 18 meters b) 16 metres c) 8 metres d) 6 metres

101
SESSION –10, 11AND12

AIM
 Motion of a body under gravity

Motion of Body under Gravity (Free Fall).


The force of attraction of earth on bodies is called force of gravity. Acceleration produced in the body
by the force of gravity, is called acceleration due to gravity. It is represented by the symbol g.
In the absence of air resistance, it is found that all bodies (irrespective of the size, weight or
composition) fall with the same acceleration near the surface of the earth. This motion of a body falling
towards the earth from a small altitude (h << R) is called free fall.
An ideal one-dimensional motion under gravity in which air resistance and the small changes in
acceleration with height are neglected.
(1) If a body dropped from some height (initial velocity zero)
(i) Equation of motion: Taking initial position as origin and direction of motion (i.e., downward
direction) as a positive, here we have

u=0
2h v
t  
g g

h v  2 gh

2
v
h 
2g

u=0 [As body starts from rest]


a = +g [As acceleration is in the direction of motion]
v=gt … (i)

h = 𝑔𝑡 .... (ii)
𝜐 =2gh .... (iii)
hn= (2n+1) .... (iv)

102
(ii) Graph of distance velocity and acceleration with respect to time:

s v a

g
tan = g


t t t

(iii) As h = (1/2)gt2, i.e., h  t2, distance covered in time t, 2t, 3t, etc., will be in the ratio of 12 :
22 : 32, i.e., square of integers.
(iv) The distance covered in the nth sec, hn = 𝑔(2𝑛 − 1)So distance covered in I, II, III sec,
etc., will be in the ratio of 1: 3: 5, i.e., odd integers only.
(2) If a body is projected vertically downward with some initial velocity
Equation of motion: h = ut + gt2

𝜐 = u2+2gh hn= u+ (2n –1)


(2) If a body is projected vertically upward
(i) Equation of motion: Taking initial position as origin and direction of motion
(i.e., vertically up) as positive
a=–g [As acceleration is downwards while motion upwards]
So, if the body is projected with velocity u and after time t it reaches up to height h then ,

v=0

2h u
h t  
g g
u
u  2 gh

2
u
h
2g

𝜐 = 𝑢 − 𝑔𝑡 h = ut– gt2𝜐 = u2–2gh hn= u – (2n –1)

103
(ii) For maximum height v = 0 So from above equation
u = gt, h = gt2 and 𝑢 = 2𝑔ℎ
(iii) Graph of distance, velocity and acceleration with respect to time (for maximum height) :

s v a
(u2/2g)

+ +
(u/g) (2u/g)
O t O t

– g
(u/g) –v –a
t

It is clear that both quantities do not depend upon the mass of the body or we can say that in absence of
air resistance, all bodies fall on the surface of the earth with the same rate.
(4) In case of motion under gravity for a given body, mass, acceleration, and mechanical energy remain
constant while speed, velocity, momentum, kinetic energy and potential energy change.
(5) The motion is independent of the mass of the body, as in any equation of motion, mass is not involved.
That is why a heavy and light body when released from the same height, reach the ground
simultaneously and with same velocity i.e., t = (2ℎ/𝑔)and v = 2𝑔ℎ
(6) In case of motion under gravity time taken to go up is equal to the time taken to fall down through the
same distance. Time of descent (t1) = time of ascent (t2) = u/g

Total time of flight T = t1 + t2 =


(7) In case of motion under gravity, the speed with which a body is projected up is equal to the speed with
which it comes back to the point of projection. As well as the magnitude of velocity at any point on the
path is same whether the body is moving in upwards or downward direction.
(8) A ball is dropped from a building of height h and it reaches after t seconds on earth. From the same
building if two ball are thrown (one upwards and other downwards) with the same velocity u and they
reach the earth surface after t1 and t2 seconds respectively then
t = √𝑡 𝑡

u=0
u
u

t1 t2 t

104
(9) A body is thrown vertically upwards. If air resistance is to be taken into account, then the time of ascent
is less than the time of descent. t2> t1

Let u is the initial velocity of body then time of ascent 𝑡 = and h =


( )
where g is acceleration due to gravity and a is retardation by air resistance and for upward motion both
will work vertically downward.
For downward motion a and g will work in opposite direction because a always work in direction
opposite to motion and g always work vertically downward.

So h = (g-a)𝑡 ⟹ ( )
= (𝑔 − 𝑎)𝑡 ⟹ t2=
( )( )
Comparing t1 and t2 we can say that t2> t1 since (g + a) > (g - a)
(10) A particle is dropped vertically from rest from a height. The time taken by it to fall through
successive distance of 1m each will then be in the ratio of the difference in the square roots of the
integer’s i.e.

u=0

1m t1  1

1m t2  2  1

1m t3  3  2

1m
t4  4  3

√1,(2–√1),(√1–√1)…….(√4 − √3),…….

CLASS EXERCISE SOLVED

Problem1. If a body is thrown up with the velocity of 15 m/s then maximum height attained by the body is
(g = 10 m/s2)
a) 11.25 m b) 16.2 m c) 24.5 m d) 7.62 m

105
( )
Solution: (a) Hmax= = = 11.25𝑚
×

Problem2. A body falls from rest in the gravitational field of the earth. The distance travelled in the fifth
second of its motion is
a) 25m b) 45m c) 90m d) 125m
𝒈
Solution (b) hn= (2n–1) ⟹ ℎ5𝑡ℎ = (2 × 5 − 1)= 45m
𝟐

Problem3. If a ball is thrown vertically upwards with speed u, the distance covered during the last t seconds
of its ascent is
a) 𝑔𝑡 b) ut– 𝑔𝑡 c) (𝑢 − 𝑔𝑡)𝑡 d) ut

Solution: (a) If ball is thrown with velocity u, then time of flight =

h t sec

u 
  t  sec
g 

velocity after − 𝑡 𝑠𝑒𝑐: 𝑣 = u– − 𝑡 =gt


So, distance in last ’t’ sec: 0 = (𝑔𝑡) − 2(𝑔)ℎ.
h = 𝑔𝑡 .

Problem4. A man throws balls with the same speed vertically upwards one after the other at an interval of 2
seconds. What should be the speed of the throw so that more than two balls are in the sky at any time
(Given g = 9.8 m/s2)
(a) At least 0.8 m/s (b) Any speed less than 19.6 m/s
(c) Only with speed 19.6 m/s (d) More than 19.6 m/s
Solution: (d) Interval of ball throw = 2 sec.
If we want that minimum three (more than two) ball remain in air then time of flight of first ball must be
greater than 4 sec. i.e. T > 4 sec or > 4 sec ⟹ 𝑢 > 19.6 m/s

106
It is clear that for First ball will just strike the ground (in sky), second ball will be at highest point (in
sky), and third ball will be at point of projection or on ground (not in sky).

Problem5. A man drops a ball downside from the roof of a tower of height 400 meters. At the same time
another ball is thrown upside with a velocity 50 meter/sec. from the surface of the tower, then they will
meet at which height from the surface of the tower
(a) 100 meters (b) 320 meters (c)80 meters (d) 240 meters
Solution: (c) Let both balls meet at point P after time t.

h1
P
400 m

h2
B

The distance travelled by ball A (h1) = 𝑔𝑡 ....... (i)

The distance travelled by ball B (h2) = 𝑢𝑡 − 𝑔𝑡 .....(ii)


By adding (i) and (ii) = 400 (Given h = h1 +h2 = 400.
∴ t = 400/50 = 8 sec and h1 = 320 m, h2 = 80m
Problem6. A very large number of balls are thrown vertically upwards in quick succession in such a way
that the next ball is thrown when the previous one is at the maximum height. If the maximum height is
5m, the number of ball thrown per minute is (take g = 10ms–2)
a) 120 b) 80 c) 60 d) 40
Solution: (c) Maximum height of ball = 5m, So velocity of projection
⟹u = 2𝑔ℎ = √2 × 10 × 5 = 10 𝑚/𝑠
time interval between two balls (time of ascent) = = 1𝑠𝑒𝑐 = min.
So no. of ball thrown per min = 60

Problem7. A particle is thrown vertically upwards. If its velocity at half of the maximum height is 10 m/s,
then maximum height attained by it is (Take m/s2)
a) 8 m b) 10 m c) 12 m d) 16 m

Solution: (b) Let particle thrown with velocity u and its maximum height is H then H =
When particle is at a height H/2, then its speed is 10 m/s

107
From equation 𝑣 = 𝑢 − 2𝑔ℎ (10)2 = u2-2g = 𝑢 − 2𝑔 ⟹ 𝑢 = 200

Maximum height H = = = 10m


×

Problem8. A stone is shot straight upward with a speed of 20 m/sec from a tower 200 m high. The speed
with which it strikes the ground is approximately
a) 60 m/sec b) 65 m/sec c) 70 m/sec d) 75 m/sec
Solution: (b) Speed of stone in a vertically upward direction is 20 m/s. So for vertical downward motion we
will consider u = -20 m/s
𝑣 − 𝑢 + 2𝑔ℎ = (−20) + 2 × 10 × 100 ⟹ 𝑣 = 65 𝑚/𝑠
Problem9. A body freely falling from the rest has a velocity 'v' after it falls through a height 'h'. The
distance it has to fall down for its velocity to become double, is
a) 2h b) 4h c) 6h d) 8h
Solution: (b) Let at point A initial velocity of body is equal to zero

u=0
A

h
x
B v

C 2v

For path AB: 𝑣 = 0 + 2𝑔ℎ ------ (i)


For path AC:(2𝑣) = 0 + 2𝑔𝑥 ⟹ 4𝑣 = 2𝑔𝑥
Solving (i) and (ii) x = 4h
Problem10. A body sliding on a smooth inclined plane requires 4 seconds to reach the bottom starting from
rest at the top. How much time does it take to cover one-fourth distance starting from rest at the top

a) 1 s b) 2 s c) 4 s d) 16 s
Solution: (b) S= 𝑎𝑡 ⟹ 𝑡√𝑠(As a = constant)

108
𝑡 𝑠 1 𝑡 4
= = = ⟹ 𝑡 = = = 2𝑠
𝑡 𝑠 𝑠 2 2 2

Problem11. A stone dropped from a building of height h and it reaches after t seconds on earth. From the
same building if two stones are thrown (one upwards and other downwards) with the same velocity u
and they reach the earth surface after and seconds respectively, then
a) t = 𝑡 − 𝑡 b) t = c) t = √𝑡 𝑡 d) t =𝑡 𝑡

Solution: (c) For first case of dropping h = 𝑔𝑡

For second case of downward throwing h = –ut+ 𝑔𝑡 = 𝑔𝑡

⟹ −𝑢𝑡 = 𝑔(𝑡 − 𝑡 ) --------- (i)

For third case of upward throwing h = u𝑡 + 𝑔𝑡 = 𝑔𝑡


⟹𝑢𝑡 = 𝑔(𝑡 − 𝑡 ) --------- (ii)

On solving these two equations: − = ⟹ 𝑡 = √𝑡 𝑡 .

Problem12. By which velocity a ball be projected vertically downward so that the distance covered by it in
5th second is twice the distance it covers in its 6th second ( g = 10 m/s2)
a) 58.8 m/s b) 49 m/s c) 65 m/s d) 19.6 m/s
Solution: (c) By formula
hn = u + 𝑔(2𝑛 − 1)
10 10
⟹𝑢− [2 × 5 − 1] = 2{𝑢 − [2 × 6 − 1]}
2 2
⟹ 𝑢 − 45 = 2 × (𝑢 − 55) ⟹ 𝑢 = 65 𝑚/𝑠

Problem13. Water drops fall at regular intervals from a tap which is 5 m above the ground. The third drop
is leaving the tap at the instant the first drop touches the ground. How far above the ground is the
second drop at that instant
a) 2.50 m b) 3.75 m c) 4.00 m d) 1.25 m

Solution: (b) Let the interval be t then from question


For first drop 𝑔(2𝑡) = 5 ........ (ii)

109
For second drop x = x = 𝑔𝑡 ------------ (ii)

By solving (i) and (ii) 𝑥 = and hence required height h =5– = 3.75m

Problem14. A balloon is at a height of 81 m and is ascending upwards with a velocity of A body of 2 kg


weight is dropped from it. If the body will reach the surface of the earth in
a) 1.5 s b) 4.025 s c) 5.4 s d) 6.75 s
Solution: (c) As the balloon is going up we will take initial velocity of falling body
By applying h = ut+ 𝑔𝑡 81 = –12t+ (10)𝑡 ⟹ 5𝑡 − 12𝑡 − 81 = 0
±√ ±√
⟹t= = ≈ 5.4 sec

Problem15. A particle is dropped under gravity from rest from a height h (g= 9.8m/s2) and it travels a
distance 9h/25 in the last second, the height h is
a) 100 m b) 122.5 m c) 145 m d) 167.5 m
Solution: (b) Distance travelled in sec = 𝑔𝑛 = h ........... (1)

Distance travelled in nth sec = (2𝑛 − 1) = ........... (ii)


Solving (i) and (ii) we get. h = 122.5m

Problem16. A stone thrown upward with a speed u from the top of the tower reaches the ground with a
velocity 3u. The height of the tower is
a) 3𝑢 /𝑔 b) 4𝑢 /𝑔 c) 6𝑢 /𝑔 d) 9𝑢 /𝑔

By applying 𝑣 = 𝑢 + 2𝑔ℎ(3𝑢) = (−𝑢) + 2𝑔ℎ h =

Problem17. A stone dropped from the top of the tower touches the ground in 4 sec. The height of the tower
is about
a) 80 m b) 40 m c) 20 m d) 160 m
Solution (a) h = 𝑔𝑡 = × 10 × 4 = 80 𝑚

Problem18. A body is released from a great height and falls freely towards the earth. Another body is
released from the same height exactly one second later. The separation between the two bodies, two
seconds after the release of the second body is
a) 4.9 m b) 9.8 m c) 19.6 m d) 24.5 m
Solution: (d) The separation between two bodies, two second after the release of second body is given by:
s= 𝑔(𝑡 − 𝑡 ) × 9.8 × (3 − 2 ) = 24.5 m

110
CLASS EXERCISE (A) :

1] In the last second of free fall, a body covered 3/4 of its total path. Then the height from
which the body is released will be
a) 4.9m b) 9.8m c) 19.6m d) 39.2m
2] A freely falling body travels a distance X in the nth second. In the next second if it travels a distance,
Y. Then
a) X+Y=g b) X – Y=g c) Y – X =g d) X = Y/g
3] A body is released from height h above the ground which takes ‘t’ seconds to reach the
ground. The position of the body after t/2 seconds is
3ℎ
a) above the ground b) above the ground

c) above the ground d) Depends upon the size of the body

4] The average velocity of freely falling body is 21 ms-1. Then it is released from a height equal to
a) 5m b) 90m c) 30m d) 60m
5] A window is 0.5m high. A stone is released from a height 0.4m above the top of the window. The time
taken by the stone to cross the window is
a) 4 s b) 3/7 s c) 1/7 s d) 2/7 s
6] A stone is dropped into a well and sound of the splash is heard 3 s later. If the depth of well is 44.1m

find the velocity of sound in air (g = 9.8ms-2)


a) 252.6ms-1 b) 352.8ms-1 c) 300ms-1 d) 350ms-1
7] A body slides down a smooth inclined plane of inclination 300 with horizontal and length 19.6m
starting from rest at the top. Find the speed and time to reach the bottom. (g = 9.8ms-2)
a) 2.5s, 15ms-1 b) 3.83s, 15.4ms-1 c) 2.83s, 13.87ms-1 d) None
8] A stone falls from the top of a tower in 8s. How much time will it take to cover the first quarter of
distance starting from top?
a) 2s b) 3s c) 4s d) 8s

HOME EXERCISE (A):

1] A body released from certain height above the ground falls through a distance h in the first & seconds.
In the next second, it falls through a distance of
a) 2𝑔ℎ b) g/2 c) h +g d) 2𝑔ℎ+g/2
2] In the last second of travel a freely falling body covers th of total distance in the last second of its
falls. The height from which it is falling is
a) 100m b) 122.5m c) 200m d) 400m

111
3] Total distance travelled in the last-second of its journey equals the distance covered by it in first three
seconds of motion. The time for which the stone is in air is
a) 5s b) 12s c) 15s d) 8s
4] A stone is dropped from a height 10cm above the top of window of 80cm high ( top to bottom of
window) Then time taken by the stone to cross from the top to bottom of window is
a) s b) s c) s d) s

5] The average velocity of a freely falling body is 7ms-1. Then it is released from a height equal to
a)5m b) 10m c) 20m d) None
6] A body falling for 2 seconds covers a distance y equal to that covered in the next second.
If g = 10ms-2; the value of y is
a) 20m b) 10m c) 30m d) 60m
7] The ratio of distances travelled by a freely falling body in first 1second, 2seconds, 3 seconds
respectively is
a) 1: 3: 5 b) 1: 4: 9 c) 1: 2: 3 d) None
8] The ratio of distances travelled by a freely falling body in 1st second, 2nd second and 3rd second of
travel is
a) 1: 2: 3 b) 1: 4: 9 c) 1: 3: 5 d) None
9] If velocity acquired by a freely falling body is x, falling from a height h; the velocity acquired if it falls
from a height 2h is
a) 8x b) 4x c) 2x d) x√2

10] If a freely falling body falls from heights 44.1m, 78.4m, 122.5m respectively, the ratio of times of
descent are
a) 1: 2: 3 b) 3: 4: 5 c) 1: 3: 5 d) None

CLASS EXERCISE (B):

1] Two bodies one held 10m vertically above the other are released simultaneously. After falling freely
for 3 seconds under gravity, their relative separation is
a) 10m b) 5m c) 1m d) none
2] After falling through the first h metres, a freely falling body acquires velocity ‘V’. After falling through
the next h metres, velocity acquired by it would be
a) 2 V b) √2 𝑉 c) V/√2 d) 4 V
3] A body is released from height ‘h’ above the ground. Exactly at the midway if g vanishes suddenly, its
total time of fall is

a) b) 2 c) d)

112
4] A parachutist after bailing out (drops out) falls a distance of 19.6m after which the parachute opens and
he decelerates downwards at 1.1ms-2. He reaches the ground with a speed of 2ms-1. Find total time
taken and the height from the ground where parachutist bails out.
a) 20s, 200m b) 40s, 300m c) 18s, 192.4m d) 16s, 172.8m
5] A ball is dropped from a height. Another ball is dropped from the same height after 2s. Their separation
after 2 more second is
a) g b) 2g c) 4g d) 6g
6] A stone is dropped into water from a bridge 44.1m above the water. Another stone is thrown vertically
downwards 1second later. Both strike the water simultaneously. Then initial speed of the second stone
is
a) 24.5ms-1 b) 49 ms-1 c) 9.8 ms-1 d) 12.25ms-1
7] A stone is dropped from a multistored building. If it crosses 2 floors in the first second of its free fall,
the numbers of floors it can cross in the 3rd second of its fall is
a) 5 b) 7 c) 10 d) 9

HOME EXERCISE (B):


1] Height from which a body is released is numerically equal to the velocity acquired finally. Then the
height is equal to
a) g b) 2g c) 4g d) 8g
2] A wooden block is dropped from the top of a cliff 100m high. Simultaneously a bullet of mass 100gm
is fired from the foot of cliff upwards with a velocity of 100ms-1. The bullet and block will meet each
other after time of
a) 10 s b) 0.5 s c) 1 s d) 7 s
3] A body is projected up with a speed u and the time taken by it is T to reach the maximum height H. Pick
out the correct statement.
1) It reaches H / 2 in T / 2 sec 2) It acquires velocity u / 2 in T / 2 sec
3) Its velocity is u / 2 at H / 2 4) same velocity at 2T
a) a is correct b) b is correct c) c is correct d) d is correct
4] A stone is dropped in river from a bridge of height 500m. If the splash of water is heard 11.47s after
dropping the stone, velocity of stone in air is (g=10ms-2)
a) 336 ms-1 b) 338ms-1 c) 340ms-1 d) 342ms-1
5] A stone is dropped from a multi stored building. It takes (3/4) s to pass the top most floor and takes 3s
to reach the ground. The total number of floors is
a) 16 b) 24 c)8 d) 12

113
Body projected vertically upward
CLASS EXERCISE (C):

1] A body is projected upwards with a velocity 98 m/s. Find:


a) The maximum height reached
b) The time taken to reach the maximum height
c) Its velocity at a height 196 m from the point of projection
d) Velocity with which it will cross down the point of projection
e) The time taken to reach back the point of projection
2] A boy throws a stone vertically up and catches it after time ‘t’ seconds. Then
maximum height reached by the stone is

a) b) c) d)

3] A body is projected vertically up with a velocity 19.6ms-1. Find the i) displacement after 4s
ii) displacement in 2nd second. iii) Velocity after 3 seconds iv) ratio of displacements in 1st and 2nd
second. iii) Velocity after 3 seconds iv) ratio of displacements in 1st and 2nd seconds respectively.
4] A body projected vertically up reaches a maximum height h in time t. The time taken
to reach half of the maximum height is
a) t/2 b) t/4 c) d) t 1 −
√ √
5] A stone projected up with a velocity U reaches two points A and B at a distance ‘h’ with velocities U/2
and U/3. The maximum height reached by the stone is
36ℎ 36ℎ
a) b) c) d)
6] A body is projected vertically upwards. If t1 and t2 be the times at which it is at a height h above the
point of projection while ascending and descending respectivley then
a) h = t1t2 b) h = 2gt1t2 c) h= 𝑔𝑡 𝑡 d) h = 𝑔𝑡 𝑡
7] In above problem the velocity of projection is
a) g(t1+t2) b) g(t1+t2) c) 2g(t1 + t2) d) 4g(t1 + t2)
8] In above problem the maximum height reached by the body is
a) g𝑡 b) g𝑡 c) g(𝑡 + 𝑡 )2 d) g(t1+t2)2
9] In above problem the velocity of the body at height is
a) gt1 b) gt2 c) 𝑔 𝑡21 𝑡22 d) 𝑔√𝑡 − 𝑡

114
HOME EXERCISE (C) :

1] An object is thrown vertically upwards. When it reaches half of its maximum height, its velocity is
10 ms---1. If g = 10 ms---2, how high does it rise?
a) 10 m b) 20 m c) 5 m d) 25 m
2] A boy throws balls vertically upwards. He throws one ball whenever the previous ball reaches the
highest point. If he throws each ball after 2 s, how high do the balls rise?
a) 19.6 m b) 9.8 m c) 4.9 m d) none
3] An object projected up vertically with a velocity of 98 ms---1 reaches a point X in its path after 4
seconds after projection. It reaches X again after another
a) 4s b) 8 s c) 12 s d) 16 s
4] When a body is projected vertically up, its velocity decreases to half of its initial velocity at a height ‘h’
above the ground. Then maximum height reaches by it is
a) 3 h b) 4 h c) 2 h d) 4h / 3
5] A particle is projected vertically upwards is at a certain height at time t1 and again at time t2.
If t1= 4 s, t2 = 6 s and g = 10 ms---2, then velocity of projection is
a) 50 ms---1 b) 40 ms---1 c) 20 ms---1 d) 10 ms---1
6] A body projected vertically up crosses a point P in its path at the end of 2 seconds and the highest point
at the end of 5 seconds. After how many seconds form the start will it reach P again?
a) 2 s b) 4 s c) 8 s d) 12 s
7] A body is projected vertically up with a speed of 20 ms---1. The distance travelled by it in 3 seconds
it(g = 10 ms---2)
a) 5 m b) 15 m c) 20 m d) 25 m
8] A body projected vertically up from the ground crosses a point P after t seconds and again after 3t/2
seconds. Maximum height reached by the body above P is
a) gt2 / 16 b) gt2 / 32 c) gt2 / 64 d) gt2 / 8
9] A ball is thrown up vertically with an initial speed of 20 ms---1. When it has reached 3 / 4 of the
maximum height, speed of that ball is (g = 10 ms---2)
a) 10m b) 5m c) 15m d) 20m
10] A body projected vertically up with velocity 4𝑔ℎ strikes a glass plate at a height ‘h’ above the ground
from which it is projected. After breaking the glass it loses half of its velocity. Then it can go up to a
further height of
a) h b) h / 2 c) h / 4 d) h / 8

115
CLASS EXERCISE (D):

1] Assume that there is tower of sufficient height H. A particle is projected from bottom of tower with
velocity u1 and another particle is dropped from top of tower with velocity u2. The time when the
particles meet is T then
a) T = b) T = c) T = d) T =
( ) ( ) ( ) ( )

2] If a particle occupies x seconds less and acquires a velocity y ms----1 more at one place than at another
in falling through the same distance. If g1 and g2 are accelerations due to gravity at those two places,
then x: y is equal to
a) g1 / g2 b) g2 / g1 c) 𝑔 𝑔 d) 1/ 𝑔 𝑔
3] A rocket is fired vertically from the ground with a resultant vertical acceleration of 10 ms---2. The fuel
is finished in one minute and the rocket continues to move up. If g = 10 ms---2, maximum height
reached by the rocket is
a) 36 Km b) 18 Km c) 9 Km d) none
4] A body is projected vertically up from ground. In the last 2 seconds of its ascent distance travelled by it
will be numerically
a) b) g c) 2g d) 4g
5] A body is projected vertically up from ground. In the last of its total journey distance travelled by it is h.
Then its initial velocity of projection is numerically
a) (h + g) b) (h _ g) c) (h + g / 2) c) (h - g / 2)
6] Water drops fall from a tap on the floor 5m below at regular intervals of time. The first drop striking
the floor when the fifth drop begins to fall. The height at which the third drop will be, from ground, at
that instant when first strikes the ground, will be (g=10 ms-2).
a) 1.25 m b) 2.15 m c) 2.75 m d) 3.75 m

HOME EXERCISE (D) :

1] A rocket is fired vertically upwards from the ground. It moves upwards with a constant acceleration of
10 ms---2 for 30 s after which the fuel is exhausted. After what time from the instant of firing will the
rocket attain the maximum height (g = 10 ms---2)
a) 30 s b) 45 s c) 60 s d) 90 s
2] A boy sees a ball going up and then back down through a window 2.45m high. If the total time the ball
us in slight for up and down motion is 1 sec. The height above the window that the ball rises is
a) 0.98 m b) 0.49 m c) 0.245 m d) 0.306 m

116
3] A body is projected vertically up from ground. Just before the last second of its ascent its velocity will
be numerically
a) b) g c) 2g d) 4g
4] A body is projected vertically up with velocity ‘u’ from ground and in the last second of its total journey
distance travelled by it is ‘h’. If that body is projected with twice the initial velocity as that of the
previous case, distance travelled by it in the last second of its total motion would be
a) 2h b) h + 2g c) 2h + g d) 2h + g/2
5] A body is projected vertically up from the ground crosses point A, B and C with velocities 3V, 2V and
V respectively. Then AB: BC =
a) 3 : 1 b) 5 : 3 c) 2 : 1 d) 3 : 5
6] When a body is projected vertically up from the ground, its velocity reduces to half of the initial
velocity after time‘t’. Then total time of its flight is
a) 2t b) 3t c) t d) 4t

Vertical projection from a height


CLASS EXERCISE (E):
1] A balloon is ascending up with a velocity of 4 ms--1. An object is dropped from it when it is at a height
of 100 m above the ground. The distance between the object and balloon after 2 seconds is
(g = 10 ms--2)
a) 10 m b) 20 m c) 30 m d) 40 m
2] A balloon rises from rest with a constant acceleration. A stone is released from it when it has risen to a
height h. The time taken by the stone to reach the ground is

a) b) c)2 d) 4

3] From the top of a tower a stone is projected vertically up with a velocity 20 ms---1. After t seconds
another3] from the top of a tower a stone is projected vertically up with a velocity 20 ms---1. After t
seconds another stone is projected vertically down with a velocity 20 ms---1 so that both the stones
reach the ground simultaneously. The t = (g = 10 ms--2)
a) 2 b) 4 c) 5 d) 8
4] A body projected vertically up with velocity u from the top of a tower reaches the foot of the tower with
velocity 2u. Then height of that tower is

a) b) c) d)
5] A rocket fired vertically upwards with constant acceleration has its engine exhausted in 10 second.
What is the maximum height reached by the rocket if its velocity at the end of 10th second
600 ms-1? [g = 10 ms---2]
a) 600 x 5 m b) 600 x 25 m c) 600 x 35 m d) 600 x 40 m

117
6] A particle is projected vertically upwards. Prove that it will be at of their greatest heights at times which
are in the ratio 1: 3
7] A stone is released from a hydrogen balloon, going upwards with velocity 12 m/s, when it is at height of
65m from the ground. Time the stone will take to reach the ground is
a) 5 sec b) 6 sec c) 7 sec d) 8 sec
8] From the top of a tower a stone is projected vertically upward. When it reaches a distance h below that
point, its velocity is double that of its velocity when it was at a height ‘h’ above the top of the tower.
Then greatest height attained by the stone above the top of the tower is
a) 2h b) h/3 c) 5h/3 d) 4h
9] A balloon rises from rest on the ground with a constant acceleration . A stone is dropped when the
balloon has rises to a height of Hm. Find the time taken by the stone to reach the ground

HOME EXERCISE (E):

1] A balloon is at a height of 81 m and it is ascending upwards with a velocity of 12 ms---1. A body of 2kg
weight is dropped from it. If g = 10 ms---2, the body will reach the surface of the earth in
a) 1.5 s b) 4.025 s c) 5.4 s d) 6.75 s
2] A stone is projected up with a velocity of 14.7 ms---1 from the top of a tower and it reaches the ground
after 5 seconds. Then height of that lower is
a) 9.8 m b) 19.6 m c) 49 m d) 98 m
3] From the top of a tower a body is projected vertically up with a velocity of 4.9 ms---1. The time after
which it will pass through the point of projection is
a) 1 s b) 2 s c) 4 s d) 6 s
4] A stone thrown vertically up with a certain velocity from the top of a lower of height h reaches the
ground with velocity which is thrice in magnitude as that of initial. Then its initial velocity is

a) 𝑔ℎ b) c) d) none

118
SESSION – 13,14AND 15

Aim: Projectile -Motion in Two Dimensions

Motion in a plane is called as motion in two dimensions e.g., projectile motion, circular motion etc. For
the analysis of such motion our reference will be made of an origin and two co-ordinate axes x and y.
Position of particle is known by knowing its co-ordinate (x, y). Velocity of particle will be resultant of
velocities in x and y direction Vx and Vy. Similarly acceleration will be in the two directions. For
analysis of such motion we analyses the motion along two axes independently i.e., while dealing motion
in x-direction we need not to think what is going on in y-direction and vice versa.

We have to study about projectile motion, circular motion and relative velocity under the head of
motion in two dimensions.
Projectile Motion
A projectile is a particle, which is given an initial velocity, and then moves under the action of its
weight alone. If the initial velocity is vertical, the particle moves in a straight line and such motion we
had already discussed in ‘motion in one dimension as motion under gravity’. Here we are going to
discuss the motion of particle which is projected obliquely near the earth’s surface. While discussing
such motion we shall suppose the motion to be within such a moderate distance from the earth’s
surface, that acceleration due to gravity may be considered to remain sensibly constant. We shall also
neglect the resistance of air and consider the motion to be in vacuum.
Important Terms used in projectile motion
When a particle is projected into air, the angle that the direction of projection makes with horizontal
plane through the point of projection is called the angle of projection; the path, which the particle
describes, is called the trajectory, the distance between the point of projection and the point where the
path meets any plane draws through the point of projection is its range; the time that elapses in air is
called as time of flight and the maximum distance above the plane during its motion is called as
maximum height attained by the projectile.
Analytical treatment of projectile motion
Consider a particle projected with a velocity u of an angle with the horizontal from earth’s surface. If
the earth did not attract a particle to itself, the particle would describe a straight line; on account of
attraction of earth, however, the particle describes a curved path. This curve will be proved later to be
always a parabola.
Let us take origin at the point of projection and x-axis and y-axis along the surface of earth and
perpendicular to it respectively as shown in figure.

119
y

u P


O O x
By the principle of physical independence of forces, the weight of the body only has effect on the
motion in vertical direction. It, therefore, has no effect on the velocity of the body in the horizontal
direction, and horizontal velocity therefore remains unaltered.

Motion in x- direction:
Motion in x - direction is motion with uniform velocity.
At t = 0, x0 = 0 and ux = u cos𝜃
Position after time t, x = x0 + uxt
⟹ x = (u cos 𝜃) t
Velocity of any time t, vx = ux ⟹vx= u cos 𝜃

Motion in y-direction:
Motion in y-direction is motion with uniformly acceleration
When, t = 0, y0 = 0, uy = u sin 𝜃 and ay = -g
∴ After time‘t’, vy = uy + ay t ⟹vy = u sin 𝜃 - gt
y = y0 + uyt + ayt2 ⟹y = uyt + ayt2

y = (u sin𝜃) t -

Also, 𝑉 = 𝑢 + 2ayy ⟹𝑉 = u2 sin2𝜃 - 2 gy


Time of Flight (T):
Time of flight is the time during which particle moves from O to O i.e., when t = T, y = 0
∴ From equation (iv)
2𝑢𝑠𝑖𝑛𝜃
O = u sin 𝜃 T- gT2 T=
Range of projectile (R):
Range is horizontal distance travelled in time T.
i.e., R = x (in time T)
∴ From equation (ii)
2𝑢𝑠𝑖𝑛𝜃
R = ucos𝜃.T = ucos𝜃
𝑠𝑖𝑛2𝜃
R=

120
Maximum height reached (H):
At the time particle reaches its maximum height velocity of particle becomes parallel to horizontal
direction i.e., vy = 0 when y = H
∴ From equation (v)

0 = u2 sin2𝜃 − 2 gH; H=
Equation of trajectory:
The path traced by a particle in motion is called trajectory and it can be known by knowing the relation
between x and y.
From equation (1) and (2) eliminating time t we get

y = x tan 𝜃 - sec2𝜃
This is trajectory of path and is equation of parabola. So we can say the path of particle is parabolic.
Velocity and direction of motion after a given time:
After time‘t’ vx = ucos 𝜃 and vy = usin 𝜃 -gt

Hence resultant velocity v = 𝑉 +𝑉


= 𝑢 𝑐𝑜𝑠 𝜃 + (𝑢𝑠𝑖𝑛 𝜃 − 𝑔𝑡)
If direction of motion makes an angle 𝛼 with horizontal
tan𝛼 = = ⟹ 𝛼 = 𝑡𝑎𝑛
𝑢𝑐𝑜𝑠𝜃

Velocity and direction of motion at a given height:


At a height ‘h’, Vx = ucos 𝜃
And Vy = 𝑢 𝑠𝑖𝑛 𝜃 − 2𝑔ℎ
Resultant velocity v = 𝑉 + 𝑉 ; v= 𝑢 − 2𝑔ℎ
Note that this is the velocity that a particle would have at height h if it is projected vertically from
ground with u.

SOME IMPORTATN POINTS REGARDING PROJECTILE MOTION OVER A


HORIZONTAL PLANE
i) For a given velocity of projection, the range of horizontal plane will be maximum when angle of
projection is 450.

We have range of projectile. R =


Therefore if we keep on increasing𝜃 range will increase and then decrease. Its value will be maximum
when sin2𝜃 is maximum i.e., = 450

Also, maximum range Rmax =

121
ii) For a given range and given initial speed of projection, there are two possible angle of projection which
are complementary angle i.e., if one is 𝜃 other will be (900-𝜃).Class Exercise solved

Problem1. The trajectory of a projectile is represented by y =√3𝑥 − 𝑔𝑥 /2. The angle of projection is
a) 30o b) 45o c) 60o d) None of these
Solution: (c) By comparing the coefficient of x in given equation with standard equation

y = x tan𝜃 − ; tan𝜃 = √3 ∴ 𝜃 = 60

Problem2. The path followed by a body projected along y-axis is given as by y = √3𝑥 – (1/2)x2 if
g = 10 m/s, then the initial velocity of projectile will be - (x and y are in m)
a) 3√10 m/s b) 2√10 m/s c) 10√3 m/s d) 10√2 m/s
Solution: (b) By comparing the coefficient of x2 in given equation with standard equation y = x tan 𝜃 −

. = Substituting 𝜃= 600 we get u =2√10m/s


𝑐𝑜

5x2
Problem3. The equation of projectile is y = 16x– The horizontal range is
a) 16 m b) 8 m c) 3.2 m d) 12.8 m
Solution: (d) Standard equation of projectile motion y = x tan 𝜃 1 −

Given equation: y = 16x – or y = 16x 1 −


/
64
By comparing above equations R = =12.8 m.

Problem4. A body of mass 2 kg has an initial velocity of 3 m/s along OE and it is subjected to a force of 4
Newton's in OF direction perpendicular to OE. The distance of the body from O after 4 seconds will be
a) 12 m b) 28 m c) 20 m d) 48 m
Solution: (c) Body moves horizontally with constant initial velocity 3 m/s upto 4 seconds
∴ 𝑥 = 𝑢𝑡 = 3 × 4 = 12m and in perpendicular direction it moves under the effect of constant force with
zero initial velocity upto 4 seconds. ∴ 𝑦 = 𝑢𝑡 + 𝑡 = 4 =16m
So its distance from O is given by d = 𝑥 +𝑦 = (12) + (16)
∴ 𝑑 = 20 𝑚

Problem5. A body starts from the origin with an acceleration of 6 m/s2 along the x-axis and 8 m/s2 along
the y-axis. Its displacement from the origin after 4 seconds will be
(a) 56 m (b) 64 m (c) 80 m (d) 128 m

122
Solution: (c) Displacement along X- axis x = 𝑢 𝑡 + 𝑎 𝑡 = × 6 × (4) = 48𝑚
Displacement along Y- axis: y = 𝑢 𝑡 + 𝑎 𝑡 = × 8 × (4) = 48𝑚
Total distance from the origin = 𝑥 + 𝑦 = (48) + (64) = 80𝑚

Problem 6.In a projectile motion, velocity at maximum height is


𝑢𝑐𝑜𝑠𝜃 𝑢𝑠𝑖𝑛𝜃
a) b) 𝑢𝑐𝑜𝑠𝜃 c) d) None of these
Solution: (b) In a projectile motion at maximum height body possess only horizontal component of velocity
i.e. u cos.

Problem7. A body is thrown at angle 30o to the horizontal with the velocity of 30 m/s. After 1 sec, its
velocity will be (in m/s) (g = 10 m/s2)
a) 10√7 b) 700√10 c) 100√7 d) √40
Solution: (a) From the formula of instantaneous velocity 𝑣 = 𝑢 + 𝑔 𝑡 − 2𝑢𝑔𝑡𝑠𝑖𝑛𝜃
𝑣= (30) + (10) × 1 − 2 × 30 × 10 × 1 × 𝑠𝑖𝑛30 = 10√7𝑚/𝑠

Problem 8.A projectile is fired at 30o to the horizontal. The vertical component of its velocity is 80 ms-1.
Its time of flight is T. What will be the velocity of the projectile at t = T/2
a) 80 ms-1 b) 80/√3ms-1 c)(80/√3) ms-1 d)40 ms-1
Solution: (b) At half of the time of flight, the position of the projectile will be at the highest point of the
parabola and at that position particle possess horizontal component of velocity only.
Given uvertical =usin𝜃 = 80 ⟹ 𝑢 = = 160𝑚/𝑠
∴ uhorizontal= ucos𝜃 = 160cos 30 = 80√3𝑚/𝑠

Problem 9.A particle is projected from point O with velocity u in a direction making an angle with the
horizontal. At any instant its position is at point P at right angles to the initial direction of projection. Its
velocity at point P is

90o P
v
u


O

a) u tan𝛼 b) u cot𝛼 c) u cosec𝛼 d) u sec𝛼

123
Solution: (b) Horizontal velocity at point ‘O’ = ucos𝛼

90o
P v sin
90o – 
u

u sin 
v

O u cos

Horizontal velocity at point ‘P’ = v sin𝛼


In projectile motion horizontal component of velocity remains constant throughout the motion
∴ v sin𝛼 = 𝑢𝑐𝑜𝑠𝛼 ⟹ 𝑣 = 𝑢𝑐𝑜𝑡𝛼

Problem 10.A particle P is projected with velocity u1 at an angle of 30o with the horizontal. Another
particle Q is thrown vertically upwards with velocity u2 from a point vertically below the highest point
of path of P. The necessary condition for the two particles to collide at the highest point is

u1
30
o u2

P Q

a) 𝑢 = 𝑢 b) 𝑢 = 2𝑢 c) 𝑢 = d) 𝑢 = 4𝑢

Solution: (b) Both particle collide at the highest point it means the vertical distance travelled by both the
particle will be equal, i.e. the vertical component of velocity of both particle will be equal
𝑢 𝑠𝑖𝑛30 = 𝑢 ⟹ = 𝑢 ∴ 𝑢 = 2𝑢

Problem 11.Two seconds after projection a projectile is travelling in a direction inclined at 30o to the
horizontal after one more sec, it is travelling horizontally, and the magnitude and direction of its
velocity are
a) 2√20 𝑚/𝑠𝑒𝑐, 60 b) 20√3 𝑚/𝑠𝑒𝑐, 60 c) 6√40 𝑚/𝑠𝑒𝑐, 30 d)40√6 𝑚/𝑠𝑒𝑐, 30
Solution: (b) Let in 2 sec body reaches upto point A and after one more sec upto point B. Total time of
ascent

124
v B
A o
30


O u cos

For a body is given 3 sec i.e. t = =3


∴ 𝑢𝑠𝑖𝑛 𝜃 = 10 × 3 = 30 ….. (i)
Horizontal component of velocity remains always constant
ucos 𝜃 = 𝑣𝑐𝑜𝑠30 ......... (ii)
For vertical upward motion between point O and A
𝑣𝑠𝑖𝑛30 = 𝑢𝑠𝑖𝑛𝜃 − 𝑔 × 2[𝑈𝑠𝑖𝑛𝑔 𝑣 = 𝑢 − 𝑔𝑡]
𝑣𝑠𝑖𝑛30 = 30 − 20[𝑎𝑠 𝑢𝑠𝑖𝑛𝜃 = 30] ∴ 𝑣 = 20𝑚/𝑠
Substituting this value in equation (ii)
Ucos𝜃 = 20𝑐𝑜𝑠30 = 10√3….. (iii)
From equation (i) and (iii) and u =20√3𝜃 = 60

Problem12. A body is projected up a smooth inclined plane (length =20√2m) with velocity u from the point
M as shown in the figure. The angle of inclination is 45o and the top is connected to a well of diameter
40 m. If the body just manages to cross the well, what is the value of v

45o

M
40 m

a) 40ms–1 b) 40√2 ms–1 c) 20 ms–1 d) 20√2 ms–1


Solution: (d) At point N angle of projection of the body will be 45°. Let velocity of projection at this point
is v. If the body just manages to cross the well then

125
v
N
R

45o
u

M 40 m

𝑠𝑖𝑛2𝜃
= 40 [as 𝜃 = 45 ]
𝑣 = 400⟹ v = 20 m/s
But we have to calculate the velocity (u) of the body at point M. For motion along the inclined plane
(from M to N) Final velocity (v) = 20 m/s, acceleration (a) = - g sin𝜃 = - g sin 45o, distance of inclined
plane (s) =

20√2(20) 𝑢 − 2 . 20√2[Using v2 = u2 + 2as] 𝑢 = 20 + 400 ⟹ 𝑢 = .

Problem 13.A projectile is fired with velocity u making angle with the horizontal. What is the change in
velocity when it is at the highest point?
a) ucos𝜃 b) u c) u sin𝜃 d) (u cos𝜃 - u)
Solution: (c) Since horizontal component of velocity remain always constant therefore only vertical
component of velocity changes. Initially vertical component usin𝜃.Finally it becomes zero. So change
in velocity = u sin𝜃

Problem14. A body of mass 0.5 kg is projected under gravity with a speed of 98 m/s at an angle of 30o
with the horizontal. The change in momentum (in magnitude) of the body is
a) 24.5 N-s b) 49.0 N-s c) 98.0 N-s d) 50.0 N-s
Solution: (b) Change in momentum between complete projectile motion = 2musin𝜃 =2×0.5×98×sin300=49
N-s.

Problem15. A particle of mass 100 g is fired with a velocity 20 m sec-1 making an angle of 30o with the
horizontal. When it rises to the highest point of its path then the change in its momentum is
a) √3 kg m sec–1 b) 1/2 kg m sec-1 c) √2 kg m sec–1 d)1 kg m sec-1
Solution: (d) Horizontal momentum remains always constant
So change in vertical momentum ( Δ𝑝⃗ ) = Final vertical momentum - Initial vertical momentum
= 0– mu sin𝜃
|ΔP| = 0.1 × 20 × 𝑠𝑖𝑛30 = 1𝑘𝑔 𝑚/𝑠𝑒𝑐

126
Problem 16.Two equal masses (m) are projected at the same angle (𝜃) from two points separated by their
range with equal velocities (v). The momentum at the point of their collision is
a) Zero b) 2 mv cos 𝜃 c) - 2 mv cos 𝜃 d) None
Solution: (a) Both masses will collide at the highest point of their trajectory with equal and opposite
momentum. So net momentum of the system will be zero.

mv cos mv cos

v v

 

Problem17. A particle of mass m is projected with velocity v making an angle of 45o with the horizontal.
The magnitude of the angular momentum of the particle about the point of projection when the particle
is at its maximum height is (where g = acceleration due to gravity)
a) Zero b) mv3/ (4√2𝑔) c) mv3/(√2𝑔) d) mv2/2g
𝑐𝑜𝑠𝜃𝑠𝑖
Solution: (b) = L = = [As 𝜃 = 45o]

Problem18. A body is projected from the ground with some angle to the horizontal. What happens to the
angular momentum about the initial position in this motion?
a) Decreases b) Increases c) Remains same d) First increases and then decreases
Solution (b)

Problem19. In case of a projectile, where is the angular momentum minimum


a) At the starting point
b) At the highest point
c) On return to the ground
d) At some location other than those mentioned above
Solution: (a)

Problem20. For a given velocity, a projectile has the same range R for two angles of projection if t1 and t2
are the times of flight in the two cases then
a) t1t2 R2 b) t1t2 R c) t1t2 1/R d) t1t2 1/R2
Solution: (b) As we know for complementary angles

127
Problem21. A body is thrown with a velocity of 9.8 m/s making an angle of 30o with the horizontal. It will
hit the ground after a time
a) 1.5 s b) 1 s c) 3 s d) 2 s
× . ×
Solution: (b) T = = = 1sec
.

Problem22. Two particles are separated at a horizontal distance x as shown in figure. They are projected at
the same time as shown in figure with different initial speed. The time after which the horizontal
distance between the particles become zero is
a) u/2x b) x/ u c) 2u / x d) u /x
Solution: (b) Let x1 and x2 are the horizontal distances travelled by particle A and B respectively in time t.
x1= . 𝑐𝑜𝑠30 × 𝑡............ (i) and x2= 𝑢𝑐𝑜𝑠60 × 𝑡 .............. (ii)

x1+x2= . 𝑐𝑜𝑠30 × 𝑡 + 𝑢𝑐𝑜𝑠60 × 𝑡 = 𝑢𝑡 ∴ 𝑡 = 𝑥/𝑢

Problem23. A particle is projected from a point O with a velocity u in a direction making an angle upward
with the horizontal. After some time at point P it is moving at right angle with its initial direction of
projection. The time of flight from O to P is
𝑢𝑠𝑒𝑐𝛼
a) b) c) d)
Solution: (b) When body projected with initial velocity by making angle 𝛼 with the horizontal. Then after
time t, (at point P) its direction is perpendicular to.

90o

u P (90 – )
u sin

 v cos v
O u cos

Magnitude of velocity at point P is given by 𝑣 = 𝑢𝑐𝑜𝑡𝛼 (from sample problem no. 9).For vertical
motion: Initial velocity (at point O) = usin 𝛼
Final velocity (at point P) = –v cos 𝛼 = −𝑢𝑐𝑜𝑡𝛼 𝑐𝑜𝑠𝛼 Time of flight (from point O to P) = t
Applying first equation of motion 𝑣 = 𝑢 − 𝑔𝑡 − 𝑢 𝑐𝑜𝑡𝛼 𝑐𝑜𝑠𝛼 =usin 𝛼 − 𝑔𝑡

𝑡= = [𝑠𝑖𝑛 𝛼 + cos 𝛼] =

128
Problem 24.A ball is projected upwards from the top of tower with a velocity 50 ms-1 making angle 30o
with the horizontal. The height of the tower is 70 m. After how many seconds from the instant of
throwing will the ball reach the ground?
a) 2.33 sec b) 5.33 sec c) 6.33 sec d) 9.33 sec
Solution: (c) Formula for calculation of time to reach the body on the ground from the tower of height 'h' (If

it is thrown vertically up with velocity u) is given by t = 1+ 1+ So we can resolve the

given velocity in vertical direction and can apply the above formula. Initial vertical component of
velocity U sin𝜃= 50 sin30 = 25 m/s.
× . ×
t= 1+ 1+ ( )
=6.33 sec
.

Problem 25.If for a given angle of projection, the horizontal range is doubled, the time of flight becomes
a) 4 times b) 2 times c) √2 times d) 1/√2times
𝑠𝑖𝑛2𝜃 2𝑢𝑠𝑖𝑛𝜃
Solution: (c)𝑅 = and T =

∴ 𝑅𝑢 and T 𝑢 (If 𝜃 and g are constant).


In the given condition to make range double, velocity must be increased upto√2 times that of previous
value. So automatically time of flight will becomes √2 times.

Problem 26.A particle is thrown with velocity u at an angle from the horizontal. Another particle is thrown
with the same velocity at an angle from the vertical. The ratio of times of flight of two particles will be
a) tan 2𝛼 : 1 b) cot 2𝛼 : 1 c) tan𝛼 : 1 d) cot 𝛼 : 1
Solution: (c) For first particles angle of projection from the horizontal is 𝛼 So T1=
For second particle angle of projection from the vertical is 𝛼 . It mean from the horizontal is (90- 𝛼)

( )
∴ T 1= = ; So ratio of time of flight = 𝑡𝑎𝑛𝛼

Problem27. The friction of the air causes vertical retardation equal to one tenth of the acceleration due to
gravity (Take g = 10 ms-2). The time of flight will be decreased by
a) 0% b) 1% c) 9% d) 11%
2𝑢𝑠𝑖𝑛𝜃
Solution: (c) T = ∴ = = =

Fractional decrease in time of flight = =


Percentage decrease = 9%

129
Problem28. A boy playing on the roof of a 10m high building throws a ball with a speed of 10 m/s at an
angle of 30o with the horizontal. How far from the throwing point will the ball be at the height of 10 m
from the ground (g = 10 m/s2, sin 30o= sin 30o =

)
a) 8.66 m b) 5.20 m c) 4.33 m d) 2.60 m
Solution :(a) Simply we have to calculate the range of projectile

u
30o

10 m 10 m

( ) ( × )
R= = R=5 √3

Problem29. Which of the following sets of factors will affect the horizontal distance covered by an athlete
in a long-jump event?
a) Speed before he jumps and his weight
b) The direction in which he leaps and the initial speed
c) The force with which he pushes the ground and his speed
d) The direction in which he leaps and the weight
( ) × ( )
Solution: (b) Because range =

Problem30. For a projectile, the ratio of maximum height reached to the square of flight time is
(g = 10 ms-2)
a) 5 : 4 b) 5 : 2 c) 5 : 1 d)10 : 1

/
Solution: (a) H = and T= ∴ = = = =
/

Problem31. A cricketer can throw a ball to a maximum horizontal distance of 100 m. The speed with
which he throws the ball is (to the nearest integer)
a) 30 ms-1 b) 42 ms-1 c) 32 ms-1 d) 35 ms-1

Solution: (c)Rmax = = =100 (when 𝜃= 450)

u = √1000 =31.62 m/s.

130
Problem32. If two bodies are projected at 30o and 60o respectively, with the same velocity, then
a) Their ranges are same b) Their heights are same
c) Their times of flight are same d) All of these
Solution: (a) Because these are complementary angles.

Problem33. Figure shows four paths for a kicked football. Ignoring the effects of air on the flight, rank the
paths according to initial horizontal velocity component, highest first

1 2 3 4
0 x

a) 1, 2, 3, 4 b) 2, 3, 4, 1 c) 3, 4, 1, 2 d) 4, 3, 2, 1
Solution: (d) Range horizontal component of velocity. Graph 4 shows maximum range, so football
possesses maximum horizontal velocity in this case.

Problem34. Four bodies P, Q, R and S are projected with equal velocities having angles of projection 15o,
30o, 45o and 60o with the horizontal respectively. The body having shortest range is
a) P b) Q c) R d) S
Solution: (a) Range of projectile will be minimum for that angle which is farthest from 45°.

Problem35. A particle covers 50 m distance when projected with an initial speed. On the same surface it
will cover a distance, when projected with double the initial speed
a) 100 m b) 150 m c) 200 m d) 250 m
𝑠𝑖𝑛2𝜃
Solution: (c) R = ∴ 𝑅𝑢 so = = ⟹ 𝑅 = 4𝑅 = 4
50 = 200 m

Problem36. A bullet is fired from a canon with velocity 500 m/s. If the angle of projection is 15o and
g = 10 m/s2. Then the range is
a) 25 × 103 m b)12.5 × 103 m c) 50 × 102 m d) 25 × 102 m
𝑠𝑖𝑛2𝜃 ( ) ( × )
Solution: (b) Range (R) = = = 12500m = 12.5× 103 m

131
Problem37. A projectile thrown with a speed v at an angle has a range R on the surface of earth. For same
v and, its range on the surface of moon will be
a) R/6 b) 6 R c) R/36 d) 36 R
𝑠𝑖𝑛2𝜃
Solution: (b) R = ∴ 𝑅

= =6 ∵ 𝑔 = 𝑔

∴ 𝑅𝑀𝑜𝑜𝑛 = 6𝑅 = 6𝑅

Problem38. A projectile is thrown into space so as to have maximum horizontal range R. Taking the point
of projection as origin, the co-ordinates of the point where the speed of the particle is minimum are
(a) (R, R) b) 𝑅, c) , d) 𝑅,
Solution: (c) For maximum horizontal Range 𝜃 = 45

R/4
o
45
X
R/2

From R = 4H cot𝜃 [As 𝜃= 45o, for maximum range.] Speed of the particle will be minimum at the
highest point of parabola. So the co-ordinate of the highest point will be (R/2, R/4)

Problem 39. The speed of a projectile at the highest point becomes times its initial speed. The horizontal

range of the projectile will be

a) b) c) d)

Solution: (a) Velocity at the highest point is given by ucos𝜃 = (given) 𝜃 = 45o

Horizontal range

132
Problem40. A large number of bullets are fired in all directions with same speed u. What is the maximum
area on the ground on which these bullets will spread?

a) 𝜋 b) 𝜋 c)𝜋 d) 𝜋
Solution: (b) The maximum area will be equal to area of the circle with radius equal to the maximum range
of projectile Maximum area

𝜋𝑟 = 𝜋 =𝜋
[As r= Rmax= 𝑢 / 𝑔 for = 450]

Problem41.A projectile is projected with initial velocity (6𝚤̂ + 8𝚥̂) m/sec. If g = 10 ms-2, then horizontal
range is
a) 4.8 metre b) 9.6 metre c) 9.2 metre d) 14.0 metre
Solution: (b) Initial velocity = (6𝚤̂ + 8𝚥̂) m/sec (given)
Magnitude of velocity of projection= 𝑢 + 𝑢 = √6 + 8 = 10 m/s
Angle of projection
( ) × × ×
tan𝜃 = = = = =9.6 meter

Problem42. A projectile thrown with an initial speed u and angle of projection 15o to the horizontal has a
range R. If the same projectile is thrown at an angle of 45o to the horizontal with speed 2u, its range
will be
a) 12 R b) 3 R c) 8 R d) 4 R
𝑠𝑖𝑛2𝜃
Solution: (c) R = ∴ 𝑅𝑢 𝑠𝑖𝑛2𝜃 =
2𝑢 sin 90
⟹𝑅 =𝑅 = 8𝑅
𝑢 sin 30

Problem 43.The velocity at the maximum height of a projectile is half of its initial velocity of projection u.
Its range on the horizontal plane is
a) √3𝑢 /2𝑔 b) 𝑢 /3𝑔 c) 3𝑢 /2𝑔 d) 3𝑢 /𝑔
Solution: (a) If the velocity of projection is u then at the highest point body posses only ucos𝜃= (given) ∴
( × ) √
𝜃 = 60 Now R = =

133
Problem 44.A projectile is thrown from a point in a horizontal place such that its horizontal and vertical
velocity component is 9.8 m/s and 19.6 m/s respectively. Its horizontal range is
a) 4.9 m b) 9.8 m c) 19.6 m d) 39.2 m
× . × .
Solution: (d) We know R = = = 39.2 m
.
Where ux = horizontal component of initial velocity, uy = vertical component of initial velocity.

Problem45. A particle is projected with a velocity v such that its range on the horizontal plane is twice the
greatest height attained by it. The range of the projectile is (where g is acceleration due to gravity)

a) b) c) d)

Solution: (a) We know R =4Hcot𝜃 2Hcot𝜃 = 4𝐻𝑐𝑜𝑡𝜃 cot𝜃 = sin𝜃 =


cos𝜃 = [𝐴𝑠 𝑅 = 2𝐻 𝑔𝑖𝑣𝑒𝑛]



. . . .
√ √
Range = = =

Problem46.The range R of projectile is same when its maximum heights are h1 and h2. What is the relation
between R and h1 and h2
a) R= ℎ ℎ b) R= 2ℎ ℎ c) R= 2 ℎ ℎ d) R= 4 ℎ ℎ

Solution: (d) For equal ranges body should be projected with angle 𝜃 or (90 − 𝜃 ) from the horizontal.

And for these angles: ℎ = = and ℎ = by multiplication of both height:

ℎ ℎ = = 16ℎ ℎ = 𝑅 ⟹ 𝑅 = 4 ℎ ℎ

Problem47. A grasshopper can jump maximum distance 1.6 m. It spends negligible time on the ground.
How far can it go in 10 seconds?
a) 5√2𝑚 b) 10√2𝑚 c) 20√2𝑚 d) 40√2𝑚
Solution: (c) Horizontal distance travelled by grasshopper will be maximum for 𝜃 = 45

1.6 m

134
Rmax= = 1.6 𝑚 ⟹ 𝑢 = 4 𝑚/𝑠 . Horizontal component of velocity of grasshopper
ucos𝜃 = 4 𝑐𝑜𝑠45 = 2√2𝑚/𝑠Total distance covered by it in 10
sec S = ucos𝜃 × 𝑡 = 2√2 × 10 = 20√2𝑚
Problem48. A projectile is thrown with an initial velocity of if the range of projectile is double the
maximum height reached by it then
a) a = 2b b) b = a c) b = 2a d) b = 4a
Solution: (c) Angle of projection 𝜃 = 𝑡𝑎𝑛 = 𝑡𝑎𝑛 𝑡𝑎𝑛𝜃 = .......... (i)

O T

𝜃 = 0o so v = 0
i.e., line parallel to time axis represents that the particle is at rest.
Form formula R = 4H cot𝜃 = 2𝐻 𝑐𝑜𝑡𝜃 = 𝑡𝑎𝑛𝜃 = 2(iii) [As R =2H given] From equation (i)
and (ii) b = 2a

y
Position

y2 D B

y1 
C
A

O x
t1 t2
Time

From triangle ABC tan𝜃 = = =


By comparing (i) and (ii) Velocity = tan𝜃
v = tan 𝜃
It is clear that slope of position-time graph represents the velocity of the particle.
Various position - time graphs and their interpretation

135
P

O T

𝜃 = 90 𝑠𝑜 𝑣 =  i.e., line perpendicular to time axis represents that particle is changing


its position but time does not changes it means the particle possesses infinite velocity. Practically this
is not possible.

CLASS EXERCISE (A):


1] The equation of trajectory of a projectile thrown from a point on the ground is y=(x - x2/40) m. If g=10
ms-2. The maximum height reached is
a) 6 m b) 8 m c) 10 m d) 12 m
2] A projectile is thrown with an initial velocity of (𝑎𝚤̂ + 𝑏𝚥̂)𝑚𝑠 .If the range of the projectile is twice the
maximum height reached by it, then
a) a = 2b b) b = a c) b = 2a d) b = 4a

COMPREHESION:
A harbour enemy ship is at a distance 180√3from the security cannon having a muzzle velocity of 60
m/s (g=10 m/s2)?
3] To what angle must the cannon be elevated to hit the ship?
a) 300 b) 450 c) 750 d) 900
4] What is the time of flight?
a) 6 sec b) 3 sec c) 10.4 sec d) 6.4 sec
5] How far should the shipis moved away from its initial position so that it becomes beyond the range of
the cannon
a) 120 m b) 240 m c) 180 m d) 50 m
6] The speed of a projectile at its maximum height is 6 m/s. If it stays in the air for a total time of 8
seconds, its horizontal range is
a) 12 m b) 24 m c) 36 m d) 48 m
7] A particle is projected with a velocity u. So that its horizontal range and maximum height reached are
equal. The maximum height reached is
a) 2u2/3g b) 4u2/5g c) u2/g d) 8u2/17g
8] The height y and the distance x along the horizontal for a body projected in the vertical plane are given
by y=8t – 5t2 and x=6t. Then initial velocity of the projected body is (g=10 ms-2)
10 -1
a) 8 ms-1 b) 4 ms-1 c) 10 ms-1 d) ms

136
9] The minimum and maximum velocities of a projectile are 10 ms-1 and 20ms-1 respectively. The
horizontal range and maximum height are respectively (g=10 ms-2)
a) 10√3 and 20 m b) 20√3 𝑚 and 15 m c) 20m and 15m d) 10√3 and 10 m
10] The greatest height to which a man can throw a stone is 30m. The greatest distance to which he can
throw will be
a) 30 m b) 60m c) 15m d) 35m
11] A stone is projected with velocity 147 m/s at 600 to the horizontal. After how much time will it make
450 to the horizontal?
a) 4.5 s b) 3 s c) 5.5 s d) 2.5 s
12] A body is projected with a velocity of 40 m/sec, and at an angle 300. The time it takes to be at a height
of 20m is (g=10 ms-2)
a) 6 sec b) 3 sec c) 2 sec d) none of the above

HOME EXERCISE (A) :


1] If body ‘A’ of mass M is thrown with velocity V at angle 300 to the horizontal and another body ‘B’ of
the same mass is thrown with the same speed at an angle of 600 to the horizontal , the ratio of horizontal
range of A to B will be
a) 1 : 3 b) 1 : 1 c) 1 :√3 d) √3 : 1
2] At the highest point of a projectile its velocity is half the initial velocity in magnitude. The angle of
projection from ground is
a) 300 b) 450 c) 600 d) 900
3] A projectile is thrown to have maximum horizontal range on the ground. The ratio of the horizontal and
vertical components of its initial velocity is
a) 1 : 1 b) 1 : 2 c) 2 : 1 d) 1 : √2
4] The K.E. of a projectile at the highest point of its trajectory is 75 % of its K.E. at the point of projection.
The angle of projection is
a) 450 b) 600 c) 300 d) 150
5] Two bodies are projected at angle 𝜃 and (90−𝜃) to the horizontal with the same speed. The ratio of their
times of flight is
a) 1 : 1 b) tan𝜃 : 1 c) 1 : tan 𝜃 d) tan2𝜃: 1
6] A stone is projected from the ground with a velocity of 20 m/s. Its maximum height is 5 m, then the
angle of projection is (g = 10 m/s2)
a) 50 b) 300 c) 450 d) 300
7] The speed of a projectile at its greatest height is 10 m/sec. If it stays in air for 5 sec. The horizontal
range is
a) 100 m b) 200 m c) 50 m d) 150 m

137
8] A body projected at an angle 300 reaches to a maximum height 15 m. Instead if it is projected vertically
upwards with same speed then the maximum height to which it rises is
a) 7.5 m b) 30 m c) 20 m d) 60 m
9] At the top of the trajectory of a projectile, the directions of its velocity and acceleration are
a) parallel to each other b) inclined to each other at an angle of 450
c) anti parallel to each other d) perpendicular to each other
10] The equations of motion of a projectile are given by x=36t metre and 2y=96t – 9.8t2metre.
The angle of projection is
a) Sin-1 (4/5) b) Sin-1(3/5) c) Sin-1(4/3) d) Sin-1(3/4)

CLASS EXERCISE (B) :


1] A body is projected with velocity u making an angle with the horizontal. Its velocity when it is
perpendicular to the initial velocity vector is
a) uSin𝜃 b) uCot𝜃 c) uTan𝜃 d) uCos𝜃
2] A ball is thrown with velocity 8ms-1 making an angle 600 with the horizontal. Its velocity will be
perpendicular to the direction of initial velocity of projection after a time of (g =10 ms-2)
. .
a) s b) s c) 0.6 s d) s
√ √ √
3] A particle is projected with velocity The time at which velocity become perpendicular to its initial
direction is
20 .
a) s b) s c) s d) s

4] The velocity of a particle when it is at a maximum height is times of its velocity when at half of its

greatest height. The angle of projection is


a) 300 b) 600 c) 450 d) 900
5] A particle is projected with a certain velocity. So as to have the same horizontal range ‘R’. If t1 and t2
are the times taken to reach this point in the two possible way.
a) t1/t2 = 2R/g b) t1t2 = 2R/g c) t1+t2 = 2R/g d) t1- t2 = 2R/g

6] A cannon ball has the same range R on a horizontal plane for two different angles of projection. If h1
and h2 are the maximum heights in two paths for which this is possible, then
a) R = h1h2 b) R = ℎ ℎ c) R=2 ℎ ℎ d) R= 4 ℎ ℎ

7] The horizontal range of a projectile is R= where T is its time of flight. Then angle of projection to
the horizontal is
a) 300 b) 450 b) 600 d) 900

138
8] An object is projected so that it just clears two walls of height 7.5m and with separation 50m from each
other. If the time of passing between the walls is 2.5s, the range of the projectile will be
(g =10 m/s2)
a) 60m b) 70m c) 20m d) 10m
9] A body is projected at an angle 300 with horizontal with an initial velocity 30 m/s. Angle made by its
velocity with the horizontal after 1.5 s is (g =10 ms-2).
a) 00 b) 300 c) 600 d) 900
10] A projectile is thrown at an angle of 300 with a velocity of 10m/s the change in velocity during the time
interval in which it reaches the highest point is
1) 10 m/s b) 5m/s c) 5√3 m/s d) 10√3 m/s
11] Two stones are thrown each with a velocity of g from the same point in opposite directions on the
ground. If each has the greatest horizontal range, their separation on reaching the ground
a) g b) 2g c) 4g d) 8g

HOME EXERCISE (B):


1] A body projected at angle ''with the horizontal. It remains for a time T in air. Its horizontal range is R,
then Tan=
𝑔𝑇
a) b) c) d)
2𝑅
2] A ball projected into air with initial velocity 20 m/s at an angle 300 with the horizontal then (all
quantities in S.I. system)
List – I List – II
A) Angle between initial velocity and acceleration P) 20√3
B) Velocity at maximum height Q) 1200
C) Maximum height R) 5
D) Range S) 10√3
a) A - Q; B - R; C- S; D -P b) A - P; B - Q; C-R; D -S
c) A-R; B-Q; C-S; D-P d) A- S; B-P; C-R; D-Q
3] If body ‘A’ of mass M is thrown with velocity V at angle 300 to the horizontal and another body ‘B’ of
the same mass is thrown with the same speed at an angle of 600 to the horizontal, the ratio of horizontal
range of A to B will be
a) 1 : 3 b) 1 : 1 c) 1 : √3 d) √3: 1
4] Two bodies are thrown from the same point with the same velocity of 50 ms-1. If their angles of
projection are complimentary angles and the difference of maximum heights is 30m, their maximum
heights (g=10 ms-2)
a) 50m and 80m b) 47.5m and 77.5m c) 30m and 60m d) 25m and 55m

139
5] The speed of a projectile V reduces to 50%, on reaching the highest point above the ground. Then its
range on the horizontal plane is

a) b) c) d)

6] A body is projected with a velocity of 40 m/sec, and at an angle 300. The time it takes to be at a height
of 20 m is (g = 10 ms---2)
a) 6 sec b) 3 sec c) 2 sec d) 1414 m/s
8] A person throws a stone with a velocity 10 m / sec at angle tan 𝜃 = 3 / 4 to horizontal from a point at
distance 8 m from the wall. The stone just passes over the wall. The height of the wall is
(g = 10 ms---2)
a) 2 m b) 1 m c) 4 m d) none
9] If a projectile at 300 to the horizontal reaches the ground two seconds after its projection, the change in
velocity in that interval of time is
a) 9.8 ms---1 b) 14.7 ms---1 c) 19.6 ms---1 d) 29.4 ms---1
10] A particle is projected with an initial velocity of 200 ms---1 in a direction which makes angle 300 with
the vertical. The horizontal distance travelled by the particle in 3 seconds is
a) 300 m b) 200 m c) 500 m d) 400 m
11] A person throws a bottle into a dustbin at the same height as he is 2 m away at an angle of 450. The
velocity of thrown is
a) g b) 𝑔 c) 2g d) 2𝑔

CLASS EXERCISE (C) :

1] From a point on the ground at a distance 2m from the foot of a vertical wall a ball is thrown at an angle
of 450 which just clears the top of the wall and afterwards strikes the ground at a distance 4m on the
other side. Then height of the wall is
a) m b) m c) 𝑚 d) 𝑚
2] A shell is fired vertically upwards with a speed of V1 from the deck of a ship traveling at a speed of V2.
A person on the shore observe the motion of the shell as parabola, whose horizontal range is given by
a) 2𝑉 𝑉 /𝑔 b) 2𝑉 𝑉 /𝑔 c) 2𝑉 𝑉 /𝑔 d) 2𝑉 𝑉 /𝑔
3] If the acceleration due to gravity increases by 1%, the range of a projectile
a) increases by 1% b) decreases by 1% c) increases by 2% d) decreases by 2%
4] A particle is thrown with a speed u at an angle 𝜃with the horizontal. When the particle makes an angle
with the horizontal, its speed changes to v
a) v = uCos𝜃 b) v= uCos𝜃Cos𝛼 c) v = uCos𝜃Sec𝛼 d) v = uSec𝜃Cos𝛼

140
5] A particles moves in the xy plane with constant acceleration a directed along the negative
y-axis. The equation of motion of the particle has the form y = 𝛼𝑥 − 𝛽𝑥 ,where𝛼 𝑎𝑛𝑑 𝛽 are positive
constants. The velocity of the particle at the origin of coordinates is:

a) 𝑎 b) 𝑎 c) 𝑎 d) 𝑎

6] A projectile is thrown with velocity u making angle ‘𝜃’ with vertical. It just crosses the tops of two
poles each of height h after 1 second and 3 second respectively. The maximum height of projectile is
a) 9.8 m b) 19.6 m c) 39.2 m d) 4.9 m
u Q
P

𝜃
h
h

7] A projectile is thrown at an angle such that it is just able to cross a vertical wall as shown in the figure.
The angle 𝜃 at which the projectile is thrown is given by [H is the maximum height]

√3
a) 𝑇𝑎𝑛 b) 𝑇𝑎𝑛 √3 c)𝑇𝑎𝑛 d) 𝑇𝑎𝑛−1
√ √ 2

8] In the above problem the initial velocity of projection is given by

a) b) √2𝑔𝐻𝐶𝑜𝑠𝜃 c) d)
𝑐𝑜𝑠𝜃 𝑆𝑖𝑛𝜃

141
9] Shots are fired simultaneously from the top and bottom of a vertical cliff with elevation = 300,
= 600 respectively and strike the object simultaneously at the same point. If a =30√3 m is the horizontal
distance of the object from the cliff, then the height of the cliff is

a) 30 m b) 45 m c) 60 m d) 90 m
10] If𝑉⃗ and 𝑎⃗ are velocity and acceleration of a projectile and time of flight is t0
a) 𝑉⃗ . 𝑎⃗ > 0for t < t0/2 b) 𝑉⃗ . 𝑎⃗ < 0 for t < t0/2
c) 𝑉⃗ . 𝑎⃗ 0for t < t0/2 d)𝑉⃗ . 𝑎⃗ 0 for t < t0/2
11] A ball is thrown from a height 19.6m with a speed 19.6ms-1 at an angle 300 to horizontal. Calculate the
time of flight & maximum height reached.
a) (1 + 2√5)s, 24.7m b) (1 + 2√5)s, 4.9m
c) (1–√5) s, 24.7m d) (1+√5) s, 24.7m
12] A body is projected with a velocity u at angle ∝ with horizontal from a height h. It reaches a maximum
height ‘2h’ and strikes the ground. The Range of the body is
∝ ∝ ∝ ∝
a) b) c) d)
13] A particle is thrown over a triangle from one end of a horizontal base and grazing the vertical falls one
the other end of the base. If 𝜃 and 𝜃 are the base angles of the triangle and 𝜃 is the angle of n
projection, then
a) tan𝜃 = tan 𝜃 ; tan 𝜃 b) tan𝜃 = tan 𝜃 + tan 𝜃
c) tan𝜃 = tan 𝜃 _ tan 𝜃 d) tan 𝜃 = tan (𝜃 +𝜃 )

HOME EXERCISE (C) :


1] For a projectile the ratio of maximum height reached to the square of flight time is (g = 10 ms---2)
a) 5 : 4 b) 5 : 2 c) 5 : 1 d) 10 : 1
2] The equations of motion of a projectile are given by x = 36 t metre and 2y = 96 t __ 9.8 t2metre. The
angle of projection is
a) 𝑆𝑖𝑛 b) 𝑆𝑖𝑛 c)𝑆𝑖𝑛 d) 𝑆𝑖𝑛

142
3] The horizontal and vertical displacements of a projectile at time ‘t’ are x = 36 t and y = 48t _ 4.9 t2
respectively. Initial velocity of the projectile in ms----1 is
a) 15 b) 30 c) 45 d) 60
4] A stone is thrown at an angle ‘’ to the horizontal reaches a maximum height ‘h’. The time of flight of
the stone is
a) 2ℎ𝑠𝑖𝑛𝜃/𝑔 b) 2 2ℎ𝑠𝑖𝑛𝜃/𝑔 c) 2 2ℎ/𝑔 d) 2ℎ/𝑔
5] A boy playing on the roof of a 10 m high building throws a ball with a speed of 10 m/s at an angle 300
with horizontal. How far from the throwing point will the ball be at the height of 10 m from the ground
(g = 10 m/s2)
a) 5.20 m b) 4.33 m c) 2.60 m d) 8.66 m
6] The range of a projectile thrown with an initial speed vat an angle of 150 to the horizontal is R. If it is
thrown with an initial velocity 2V at an angle of 300, then the horizontal range will be
a) 2R b) 2√3R c) 4R d) 4√3R
7] A projectile has a range R on the surface of the earth. For the same angle of projection and half of the
velocity of projection, the range on another planet is 2R. Then acceleration due to gravity on that planet
is
a) g/4 b) 4g c) g/8 d) 8g
8] A projectile is fired at an angle of 530 with the ground with a speed of 100 ms-----1. The time taken for
the inclination with the ground to become 450 is (g = 10 ms-----2) (sin 530 = 4/5)
a) 1s b) 2s c) 4s d) 1/2s
9] The equation of motion of a body moving a plane is given by y =√3𝑥 − . Then velocity of its
projection is
a) 10 ms-----1 b) 20 ms-----1 c) 5 ms-----1 d) 15 ms-----1
10] The range of a projectile when launched at an angle of projection 𝜃 is R. When launched at 2 𝜃, the
range is still R. Then 𝜃 should be
a) 𝜋/3 b) 𝜋/4 c)𝜋/6 d) 𝜋/2

143
SESSION –16

AIM: Projection on Inclined Plane.


A particle is projected from a point A on an inclined plane, which is inclined at an angle 𝛽 to the
horizon with a velocity u at an elevation . The direction of projection lines in the vertical plane through
AB, the line of the greatest slope of the plane.
Let the particle strike the plane at B so that AB is the range on the inclined plane.
u
B
 C
A
The initial velocity of projection u can be resolved into a component u cos (𝛼 -𝛽) along the plane and a
component u sin (𝛼 -𝛽) perpendicular to the plane. The acceleration due to gravity g which acts
vertically down can be resolved into components g sin𝛽 up the plane and g cos𝛽 perpendicular to the
plane. By the principle of physical independence of forces the motion along the plane may be
considered independent of the motion perpendicular to the plane. Let T be the time, which the particle
takes to go from A to B. Then in this time the distance traversed by the projectile perpendicular to the
plane is zero.
∴ 0 = u sin (𝛼 - 𝛽) T - g cos𝛽T2
( )
T=
During this time the horizontal velocity of the projectile (u cos𝛼 ) remains constant. Hence the
horizontal distance described is given by
( ) ( )
AC = u cos𝛼 T = ∴ 𝐴𝐵 = =
( )
Range on the inclined plane =

Maximum range on the inclined plane


( )
R= = [sin (2𝛼 - 𝛽) - sin𝛽]
For given values of u and , R is maximum when sin (2𝛼 − 𝛽 ) = 1
0
i.e., (2 𝛼 − 𝛽 ) = 90
0
𝛼 = (45 +𝛽 /2)

144
If Rm represents the maximum range on the inclined plane.

R = (1- sin𝛽 ); R =
m m ( )
For a given velocity of projection, it can be shown that there are two directions of projection which are
equally inclined to the direction of maximum range.

Now R = [sin (2𝛼- 𝛽) - sin𝛽]


For given values of u, and R, sin (2𝛼 − 𝛽) is constant. There are two values of (2 𝛼 − 𝛽) each less than
0
180 that can satisfy the above equation.
Let (2+𝜃 − 𝛽) and (2𝜃 − 𝛽) be the two values. Then
_ 0 0
2𝜃 - 𝛽= 180 - (2𝜃 -𝛽) ;𝜃 - 𝛽/2 = 90 - (𝜃 - 𝛽 /2)
0 0
𝜃 - (45 + 𝛽/2) = (45 +𝛽/2) - 𝜃
0
Since (45 + 𝛽/2) is the angle of projection giving the maximum range, it follows that the direction
giving maximum range bisects the angle between the two angles of projection that can give a particular
range.
The initial component of velocity perpendicular to PQ is u sin (𝛼 − 𝛽 ) and the acceleration in this
direction is (-g cos𝛽 ) .If T is the time the particle takes to go from P to Q then in time T the
displacement described in a direction perpendicular to PQ is zero.
2
0 = u sin (𝛼 − 𝛽 ). T- gcos𝛽.T
T=
If the direction of motion at the instant when the particle hits the plane be perpendicular to the plane,
then the velocity at that instant parallel to the plane must be zero.
∴u cos (𝛼 − 𝛽) - g sin𝛽T = 0
( ) ( )
=T=
∴ cot𝛽 = 2 tan (𝛼 − 𝛽 )

145
CLASS EXERCISE:
1] If time taken by the projectile to reach Q is T, then PQ is equal to

v
90
P

a) Tv Sin𝜃 b) Tv Cos𝜃 c) Tv Sec𝜃 d) Tv Tan𝜃


2] A heavy particle is projected from a point at the foot of a fixed plane, inclined at an angle 45o to the
horizontal, in the vertical plane containing the line of greatest slope through the point. If 𝜙(>450) is the
inclination to the horizontal of the initial direction of projection, for what value of tan 𝜙 will the particle
strike the plane?
i) horizontally ii) at right angle

3] Two bodies are projected from the same point with equal speeds in such directions that they both strike
the same point on a plane whose inclination is 𝛽. If be the angle of projection of the first body with the
horizontal find the ratio of their times of flight

HOME EXERCISE:
1] A particle is projected with velocity u perpendicular to the inclined plane. If range
of particle is R and time of flight of particle is T then is
u

a) u tan𝜃 b) u cot𝜃 c) u sin𝜃 d) u cos𝜃

146
SESSION – 17
AIM: Horizontal projection
Consider a body projected horizontally with a velocity u at a height h.

O Vel=u

Pa
th
h

of
bo
yd
A B
Ground

Components Components
Along x-axis Along y - axis

Initial ux = u cos 0 = u uy = u sin 0 = O


velocity
Acceleration ax = 0 ay = + g
displacement x y
Velocity at time t Vx = ux = u cos 0 = u Vy = uy + ayt
displacement after t seconds x = ux t + axt2 y = uyt +ayt2
x = ut y = gt2
Velocity at time V= u+2axx V= u+ 2ay
after a Vx = ux V = 2gy
displacement
Resultant velocity = V = 𝑉 + 𝑉

= 𝑢 + 𝑢 +𝑎 𝑡 = 𝑢 +𝑔 𝑡

= 𝑢 + (𝑢 + 2𝑎𝑦) = 𝑢 + 2𝑔𝑦
Time of descent (td): As the body reaches ground (Point B) from point of projection (Point O), the time
taken is tA.The vertical displacement is OA

y = uyt + ayt2 ⟹ ℎ = g𝑡 ⟹td =

Range (R): AB is range of body x = ux t+ axt2⟹R = ut = u

147
CLASS EXERCISE:

1] An aeroplane flying horizontally with speed 90 km/hr releases a bomb at a height of 78.4m from the
ground, when will the bomb strike the ground? [g = 10 m/s2]
a) 8 sec b) 6 sec c) 4 sec d) 10 sec
2] A body thrown horizontally with velocity 10 m/s from certain height of 10m
List – I
A) Horizontal displacement covered by it when it reaches the ground
B) Horizontal displacement covered by it when its velocity components are equal
C) Vertical component of velocity when it reaches the ground
D) Horizontal component of velocity when its velocity components are equal
List – II
P) 5
Q) 10
R) 14.14
S) 20
a) A - Q, B - S; C - R, D- P b) A- R, B-S; C - P, D-Q
c) A - R, B- Q; C - R, D- Q d) A -Q, B - S; C - R, D - P
3] A ball is dropped from the top of a tower and another ball is thrown horizontally at the same time. Then
a) first ball reaches the ground earlier b) second ball reaches the ground earlier
c) both reach the ground simultaneously d) we cannot decide

HOME EXERCISE:
1] A body is projected horizontally from the top of a hill with a velocity 9.8 ms-----1. The time elapsed
before the vertical component of velocity is twice the horizontal velocity is
a) 0.5 s b) 1s c) 2s d) 1.5s
2] A body is projected horizontally from the top of a tower with 30 ms–1. If g = 10 ms–2, the velocity of
that body after 4 seconds
a) 40 ms–1 b) 30 ms–1 c) 50 ms–1 d) 20 ms–1

3] A particle projected horizontally from the top of a table touches the ground at a distance ‘d’ from the
edge of the table. If h is the height of the table, then the velocity of projection is
√ 2𝑔 √
a) h b) h c) d) d

4] A ball thrown horizontally from the top of a tower with a speed of 12.5 ms–1 strikes the ground 75 m
away from the tower. If g = 10 ms–2, height of the tower is
a) 45 m b) 90 m c) 150 m d) 180 m

148
5] An aeroplane flying horizontally at a height of 0.49 Km with a velocity 98 ms–1 released a bomb. On
reaching the ground velocity of the bomb is
a) 49 ms–1 b) 49√2ms–1 c) 98ms–1 d) 98√2 ms–1
6] A body is projected horizontally with velocity 19.6 ms–1 from certain height, the angle made by its
direction of motion to the horizontal is
a) 900 b) 600 c) 450 d) 00
7] A bullet shot from a rifle at 25 m range states the target at a point 4.9 cm below the point at which the
barrel is pointing horizontally. Then muzzle velocity of the bullet is
a) 250 ms-----1 b) 125 ms-----1 c) 100 ms-----1 d) 50 ms-----1
8] A body is projected horizontally from certain height. After 2 seconds, its direction of motion makes 300
to the horizontal. Then its initial velocity is
a) 2g/√3 b) 2√3g c) 3𝑔/g d) none
9] A body projected horizontally with a velocity u from the top of a tower strikes the ground with a
velocity 2u. Then height of that tower is
a) 3u2 / 2g b) u2 / 2g c) u2 / g d) 3u2 / 4g
10] A body projected horizontally with a velocity V from the top of a tower of height h. touches the level
ground at a distance x from the foot of the tower. Another body is projected horizontally with velocity
2V simultaneously from the same point exactly in the opposite direction. On reaching the ground,
separation between the two bodies is
a) 2x b) x c) 3x d) 4x

Comprehension:
Two particles thrown horizontally with the velocities 4 m/s and 9 m/s from certain height in opposite
directions simultaneously, then (g=10 ms-2)
1] The time in which their velocities are perpendicular
a) 1.2 sec b) 0.6 sec c) 3.6 sec d) 7.2 sec
2] The time in which their displacements are perpendicular
a) 1.2 sec b) 0.6 sec c) 3.6 sec d) 7.2 sec
3] The vertical displacement of separation between the two particles when their velocities are
perpendicular is
a) 15.6 m b) 7.8m c) zero d) 46.8m
4] The distance of separation between the two particles when their displacements are perpendicular
a) 15.6m b) 7.8m c) zero d) 46.8m
5] When a body is projected horizontally with velocity u from certain height, it reaches the ground with
velocity 4u. Then height from which it is thrown is:
16
a) b) c) d)

149
6] A ball rolls off the top of a stair case horizontally. If each step has height h and
width x and the ball just hits the nth step, initial velocity of the ball is
𝑛𝑔𝑥 𝑛𝑔 𝑛𝑔
a) 𝑥 b) c) 𝑥 d)

7] From the top of a building 20m high, a ball is projected horizontally. If the line joining the point of
projection to the point where it hits the ground makes an angle of 450 with the horizontal, then the
initial velocity of the stone is (g=10 ms-2)
a) 5 ms-1 b) 10 ms-1 c) 15 ms-1 d) 20 ms-1
8] Two paper screens A and B are separated by a distance of 100m. A bullet pierces screen A and then
screen B. The hole in B is 10cm below in A. If the bullet is traveling horizontally at A, then its
velocity at A is
100
a) 700 ms-1 b) 350 ms-1 c) 700 m/s d) 𝑚𝑠
9] A fighter plane flying horizontally at an altitude of 2km with speed of 540 kmph passes directly
overhead an antiaircraft gun. If the gun can fire a bullet at the muzzle speed of 500 ms-1, at what angle
with the vertical the gun should fire the bullet so that the bullet hits the plane?
a) Cos-1 (3/10) b) Sin-1 (3/10) c) Tan-1 (10/3) d) 450

150
SESSION – 18AND19

AIM: RELATIVE MOTION


Motion is a combined property of the object under study and the observer. Motion is always relative,
there is no such term like absolute motion or absolute rest. Motion is always defined with respect to
an observer or reference frame.
Reference frame:
Reference frame is an axis system from which motion is observed. A clock is attached to measure
time. Reference frame can be stationary or moving. There are two types of reference frame:
(i) Inertial reference frame: A frame of reference in which Newton’s first law is valid is called as
inertial reference frame.
(ii) Non-inertial reference frame: A frame of reference in which Newton’s first law is not valid is
called as non-inertial reference frame.

Note: Earth is by definition a non-inertial reference frame because of its centripetal acceleration towards
sun. But, for small practical applications earth is assumed stationary hence, it behaves as an inertial
reference frame.

RELATIVE VELOCITY
Definition: Relative velocity of a particle (object) A with respect to B is defined as the velocity with
which A appears to move is B if considered to be at rest. In other words, it is the velocity with which
A appears to move as seen by the B considering itself to be at rest.

Relative motion along straight line –

vA = , vB =
𝑋 =𝑋 −𝑋

𝑑𝑋 𝑑𝑋
𝑉 = − 𝑉 = 𝑉𝐵 − 𝑉𝐴
𝑑𝑡 𝑑𝑡 𝐴𝐵
⟹ 𝑉 = 𝑉 − 𝑉 = 0(𝑣𝑒𝑙𝑜𝑐𝑖𝑡𝑦 𝑜𝑓 𝐴 𝑤𝑖𝑡ℎ 𝑟𝑒𝑠𝑝𝑒𝑐𝑡 𝑡𝑜 𝐴)
Note: velocity of an object w.r.t. itself is always zero.

151
CLASS EXERCISE:

Ex.1 An object A is moving with 5 m/s and B is moving with 20 m/s in the same direction. (Positive x-
axis)
(i) Find velocity of B with respect to A.
Sol. vB = 20𝚤̂ m/s Þ vA = 5𝚤̂ m/s Þ vB – vA = 15𝚤̂ m/s

(ii) Find velocity of A with respect to B


Sol. vB = 20𝚤̂ m/s, vA = 15𝚤̂ m/s Þ vAB = vA – vB = – 15𝚤̂ m/s
Note: vBA = – vAB

Ex.2 Two objects A and B are moving towards each other with velocities 10 m/s and 12 m/s respectively
a shown.

(i) Find out velocity of A with respect to B.


Sol. vAB = vA – vB = (10) – (–12) = 22 m/s towards right.

(ii) Find out velocity of B with respect to A


vBA = vB – vA = (–12) – (10) = –22 m/s towards left.

Velocity of Approach

It is the rate at which a separation between two moving particles decreases.


If separation decreases velocity of approach is positive,

Velocity of approach = 22 m/s

Velocity of approach = 15 m/s

If separation increases, velocity of approach is negative. It is mainly called velocity of separation.

152
Velocity of separation:

It is the rate with which separation between two moving object increases.
Velocity of separation = 2 m/s

Velocity of separation = 15 m/s

Illustration:
Two balls A and B are moving in the same direction with equal velocities, find out their relative
velocity.

Velocity of A with respect to B 𝑉⃗ =0

Illustration:
A and B are thrown vertically upward with velocity, 5 m/s and 10 m/s respectively
(g = 10 m/s2. Find separation between them after one second
Sol. SA = ut – gt2 = 5t – × 10 × t2

= 5 × 1 – 5 × 12 = 5 – 5= 0
SB = ut – gt2. = 10 × 1 – × 10 × 12
= 10 – 5 =5
∴ SB – SA = separation = 5m.
Alter:
By relative 𝑎⃗ =𝑎⃗ − 𝑎⃗
= (–10) – (–10) = 0
Also 𝑉⃗ = 𝑉⃗ − 𝑉⃗ = 10 – 5 = 5 m/s
∴ 𝑆⃗ (in 1 sec) = 𝑉⃗ × t = 5 × 1= 5 m
∴ Distance between A and B after 1 sec = 5 m.

153
Illustration:
A ball is thrown downwards with a speed of 20 m/s from top of a building
150 m high and simultaneously another ball is thrown vertically upwards
with a speed of 30 m/s from the foot of the building. Find the time when
both the balls will meet.
(g = 10 m/s2)
Sol. (I) S1 = 20 t + 5 t2
+ S2 = 30 t – 5 t2
____________________
150 = 50 t
⟹ t = 3 s.

(II) Relative acceleration of both is zero since both have acceleration in downward
Direction
𝑎⃗ =𝑎⃗ − 𝑎⃗ =g – g = 0
𝑉⃗ == 30 – (–20)
= 50
sBA = vBA × t
𝐵𝐴
t= = =3s
Ex.5 Two cars C1 and C2 moving in the same direction on a straight road with velocities 12 m/s and 10
m/s respectively. When the separation between the two was 200 m C2 started accelerating to avoid
collision. What is the minimum acceleration of car C2 so that they don’t collide.

Sol. By relative
𝑎⃗ =𝑎⃗ − 𝑎⃗ = 0 – a = (–a)
𝑉⃗ =𝑉⃗ − 𝑉⃗ = 12 – 10 = 2 m/s.

So by relativity we want the car to stop.


\ v2 – u2 = 2as.
Þ 0 – 22 = – 2 × a × 200 Þ a = m/s2

154
= 0.1 m/s2 = 1 cm/s2.
Minimum acceleration needed by car C2 = 1 cm/s2
RELATIVE MOTION IN LIFT:
Illustration:
A lift is moving up with acceleration a. A person inside the lift throws
the ball upwards with a velocity u relative to hand.
(a) What is the time of flight of the ball?
(b) What is the maximum height reached by the ball in the lift?
Sol. (a) 𝑎⃗ =𝑎⃗ − 𝑎⃗ = (g + a) downwards

𝑆⃗ =𝑢⃗ t + t2 0= uT – (g + a)T2

T=
( )

(b) v2 – u2 = 2 as 0 – u2 = –2(g + a) H

H=
( )

Projectile motion in a lift moving with acceleration a upwards


(1) Initial velocity = u
(2) Velocity at maximum height = u cos q
(3) T=

(4) Maximum height (H) =


( )
𝑠𝑖𝑛2𝜃
(5) Range =
( )

RELATIVE MOTION IN TWO DIMENSIONS


𝑟⃗ = position of A with respect to O
𝑟⃗ = position of B with respect to O
𝑟⃗ = position of A with respect to B.
𝑟⃗ = 𝑟⃗ − 𝑟⃗
( ⃗𝐴𝐵 ) ( ⃗ ) ( ⃗ )
∴ = – . ⟹𝑉⃗ = 𝑉⃗ − 𝑉⃗
( ⃗ ) ( ⃗ ) ( ⃗ )
= – ⟹𝑎⃗ = 𝑎⃗ − 𝑎⃗

155
Note: These formulae are not applicable for light.

Illustration:
Object A and B has velocities 10 m/s. A is moving along East while B is moving towards North
from the same point as shown. Find velocity of A relative to B (𝑉⃗ )

Sol. 𝑉⃗ = 𝑉⃗ − 𝑉⃗
∴ 𝑉⃗ = 10√2

Note: 𝑉⃗ − 𝑉⃗ = 𝑉 + 𝑉 − 2𝑉 𝑉 𝑐𝑜𝑠𝜃

Illustration:

Two particles A and B are projected in air. A is thrown horizontally, B is


thrown vertically up. What is the separation between them after 1 sec?

Sol. 𝑎⃗ =𝑎⃗ − 𝑎⃗ =0

∴ 𝑉⃗ = √10 10 = 10√2

∴𝑆 = 𝑉 t = (10√2) t = 10√2 m

Consider the situation, shown in figure

156
Ex.23 (i) Find out velocity of B with respect to A
𝑉⃗ = 𝑉⃗ − 𝑉⃗ = 20 𝚥̂ – 20𝚤̂

(ii) Find out velocity of A with respect to B


𝑉⃗ = 𝑉⃗ − 𝑉⃗ = 20 𝚥̂ – 20𝚤̂

Ex.24

(1) Find out motion of tree, bird and old man as seen by boy.
(2) Find out motion of tree, bird, boy as seen by old man
(3) Find out motion of tree, boy and old man as seen by bird.
Sol. (1) With respect to boy:
vtree = 16 m/s (←)
vbird = 12 m/s (↑)
vold man = 18 m/s (←)

(2) With respect to old man:


vBoy = 18 m/s (→)
vTree = 2 m/s (→)
vBird = 18 m/s (→) and 12 m/s (↑)
(3) With respect to Bird:
vTree = 12 m/s (↓) and 16 m/s (←)
vold man = 18 m/s (←) and 12 m/s (↓).
vBoy = 12 m/s (↓).

157
MOTION OF A TRAIN MOVING ON EQUATOR:

If a train is moving at equator on the earth’s surface with a velocity vTE relative to earth’s surface
and a point on the surface of earth with velocity vE relative to its centre, then

𝑉⃗ = 𝑉⃗ − 𝑉⃗ or𝑉⃗ = 𝑉⃗ + 𝑉⃗

So, if the train moves from west to east and if the train moves from east to west
(the direction of motion of earth on its axis) (i.e. opposite to the motion of earth)

𝑉⃗ = 𝑉⃗ + 𝑉⃗ 𝑉⃗ = 𝑉⃗𝑇𝐸 − 𝑉⃗

Relative Motion on a moving training:

If a boy is running with speed on a train moving with velocity relative to ground, the speed of the
boy relative to ground will be given by:

𝑉⃗ = 𝑉⃗ − 𝑉⃗

or 𝑉⃗ = 𝑉⃗ + 𝑉⃗

so, if the boy is running in the direction of train

vB = u + v
and if the boy is running on the train in a direction opposite to the motion of train

158
vB = u – v

RELATIVE MOTION IN RIVER FLOW:


If a man can swim relative to water with velocity 𝑉⃗ and water is following relative to ground with
velocity𝑉⃗ , velocity of man relative to ground 𝑉⃗ will be given by:
𝑉⃗ = 𝑉⃗ − 𝑉⃗ or 𝑉⃗ = 𝑉⃗ + 𝑉⃗
So, if the swimming is in the direction of flow of water,
vm = vmR + vR

and if the swimming is opposite to the flow of water,


vm = vmR – vR

Illustration:
A swimmer capable of swimming with velocity v relative to water jumps in a flowing river having
velocity u. The man swims a distance d downstream and returns back to the original position. Find
out the time taken in complete motion.

Sol. t = tdown + tup

= + = Ans.

CROSSING RIVER:
A boat or man in a river always moves in the direction of resultant velocity of velocity of boat (or
man) and velocity of river flow.
1. Shortest Time:

159
The person swims perpendicular to the river flow crossing a river: consider a river having flow
velocity 𝑉⃗ and swimmer jump into the river from a point A, from one bank of the river, in a
direction perpendicular to the direction of river current. Due to the flow velocity of river the
swimmer is drifted along the river by a distance BC and the net velocity of the swimmer will be 𝑉⃗
along the direction AC.

If we find the components of velocity of swimmer along and perpendicular to the flow these are.
y
B

Velocity along the river, vx = vR. drift = x C

Velocity perpendicular to the river, vf = vmR

d
vr
The net speed is given by vm = 𝑉 +𝑉
vmR vm
 x
A

at an angle of tan𝜃 = (downstream with the direction of flow).


Velocity of vy is used only in crossing the river, ∴time taken to cross the river is

t= = .

Velocity vx is only used to drift the motion of the swimmer in the river,
drift is along the river flow,
x = (vx) (t) or x =𝑉

Illustration:
A river 400 m wide is flowing at a rate of 2.0 m/s. A boat is sailing at a velocity of 10 m/s with
respect to the water, in a direction perpendicular to the river.
(a) Find the time taken by the boat to reach the opposite bank.
(b) How far from the point directly opposite to the starting point does the boat reach
the opposite bank.
(c) In what direction does the boat actually move.

Sol.

160
(a) time taken to cross the river
t= = = 40 s Ans.
/
(b) drift (x) = (vx)(t) = (2m/s) (40s) = 80 m Ans.
(c) Actual direction of boat,
𝜃 = tan–1 = tan–1 5, (downstream) with the river flow.

2. SHORTEST PATH:
When the person crosses the river perpendicularly (along the shortest path). It should swim
upstream making an angle q with AB such that the resultant velocity, of man must be
perpendicular to the flow of river along AB.

If we find the components of velocity of swimmer along and perpendicular to the flow, these are,
velocity along the river, vx = 0

and velocity perpendicular to river vy = 𝑉 −𝑉


The net speed is given by vm = 𝑉 −𝑉
at an angle of 90º with the river direction.

Velocity vy is used only to cross the river, therefore time to cross the river,

t= = .

and velocity vx is zero, therefore, in this case the drift (x) should be zero.
and velocity vx is zero, therefore, in this case the drift (x) should be zero.
x=0
or vx = vR – vmR sin𝜃 = 0
or vR = vmR sin𝜃

161
or 𝜃 = sin–1
Hence, to cross the river perpendicular (along the shortest path) the man should swim at an angle of
+ 𝑠𝑖𝑛 upstream from the direction of river flow.

Further, since sin𝜃< 1,


Swimmer can cross the river perpendicularly only when vmR> vRie.,
Practically it is not possible to reach at B if the river velocity (vR) is too high.

Illustration:
A man can swim at the rate of 5 km/h in still water. A river 1 km wide flows at the rate of 3 km/h.
The man wishes to swim across the river directly opposite to the starting point.
(a) Along what direction must the man swim?
(b) What should be his resultant velocity?
(c) How much time he would take to cross?
Sol. The velocity of man with respect to river vmR= 5 km/hr, this is greater than the river flow velocity,
therefore, he can cross the river directly (along the shortest path). The anlge of swim must be
𝜃 = + sin–1 = 90º + sin-1

= 90º + sin–1 = 90º + 37º = 127º, with the river flow (upstream) Ans.

(b) Resultant velocity will be vm = 𝑉 −𝑉


= √5 − 3 = 4 km/hr
along the direction perpendicular to the river flow.
(c) time taken to cross the
t= = = h = 15 min
/

Ex. The velocity of about in still water is 5 km/h it crosses 1 km wide river in 15 minutes along the
shortest possible path. Determine the velocity of water in the river in km/h
Ans. 3km/h

Illustration:
A man wishes to cross a river flowing with velocity u jumps at an anlge𝜃 with the river flow. Find
out the net velocity of the man with respect to ground if he can swim with speed v. Also find
How far from the point directly opposite to the starting point does the boat reach the opposite bank?
In what direction does the boat actually move? If the width of the river is d.
B x C'
C
162
d
u
v
 
x
A
Sol. Velocity of man = vM = √𝑢 +𝑉 + 2𝑉𝑢𝑐𝑜𝑠𝜃 Ans.

tanϕ = Ans.
𝑉𝑐𝑜𝑠𝜃
(v sin𝜃) t = d ⟹ t=

x = (u + v cos𝜃) t = (u + v cos𝜃) Ans.

Illustration: Shortest Drift (v<u)


A boat moves relative to water with a velocity which is n times less than the river flow velocity. At
what anlge to the stream direction must the boat move to minimize drifting?
Sol. In this problem, one thing should be are fully meted
that the velocity of boat is less than the river flow
velocity. In such a case, boat cannot reach the point
directly opposite to its starting point. i.e. drift can never
be zero. Thus, to minimize the drift, boat starts at an
angle 𝜃 from the normal direction up stream as shown.

Now, again if we find the components of velocity of boat along and perpendicular to the flow, these
are, velocity along the river, vx = u – v sin𝜃.
and velocity perpendicular to the river, vy = v cos𝜃.

time taken to cross the river is t = = .


𝑉𝑐𝑜𝑠𝜃
In this time, drift x = (vx)t

= (u – v sin𝜃)

or x= sec𝜃 – d tan𝜃

The drift x is minimum, when = 0,

or (sec𝜃 . tan𝜃 ) – d sec2𝜃 = 0


or sin 𝜃 = 1
or sin 𝜃 = = (as v = )

so, for minimum drift, the boat must move at an angle = sin–1 from normal direction or

163
an angle + sin–1 from stream direction.

164
SESSION – 20
AIM
 To study Rain Problems
 To find the Shortest Distance
 To Solve problems related to relative velocity

RAIN PROBLEMS:
If rain is falling vertically with a velocity and on observer is moving horizontally with velocity, the
velocity of rain relative to observer will be:
𝑉⃗ = 𝑉⃗ − 𝑉⃗ or 𝑉⃗ = 𝑉 + 𝑉
and direction 𝜃 = tan–1 with the vertical as shown in figure.

Illustration:
Rain is falling vertically and a man is moving with velocity 6 m/s. Find the angle with which
umbrella should be hold by man to avoid getting wet.

Sol.
𝑉⃗ = –10 𝑉⃗ =6
Velocity of rain with respect to man =𝑉⃗ = –10𝚥̂ – 6𝚤̂

tan𝜃 = 𝜃 = tan–1
Where 𝜃 is angle with vertical

165
Illustration:
A man moving with 5m/s observes rain falling vertically at the rate of 10 m/s. Find the speed and
direction of the rain with respect to ground.

VRM = 10 m/s, VM = 5 m/s


𝑉⃗ = 𝑉⃗ – 𝑉⃗
∴ 𝑉⃗ =𝑉⃗ – 𝑉⃗
∴ 𝑉⃗ = 5√5
tan𝜃 = , 𝜃 = tan–1
Illustration:
A man standing, observes rain falling with velocity of 20 m/s at an angle of 30º with the vertical.
(1) Find out velocity of man so that rain appears to fall vertically.
(2) Find out velocity of man so that rain again appears to fall at 30º with the vertical.
Sol. (1) 𝑉⃗ = –v 𝚤̂ (let)
𝑉⃗ = –10 𝚤̂– 10√3𝚥̂
𝑉⃗ = – (10 – v)𝚤̂ – 10√3𝚥̂
⟹ – (10 – v) = 0 (for vertical fall, horizontal component must be zero)
or v = 10 m/s Ans.
(2) 𝑉⃗ = –10 𝚤̂– 10√3𝚥̂
𝑉⃗ = –vx𝚤̂
𝑉⃗ = – (10 – vx)𝚤̂ – 10√3𝚥̂
Angle with the vertical = 30º
⟹ tan 30º =

⟹ vx = 20 m/s

166
WIND AIRPLANE:
This is very similar to boat river flow problems the only difference is that boat is replaced by also
plane and river is replaced by wind.
Thus,
velocity of aeroplane with respect to wind
𝑉⃗ = 𝑉⃗ − 𝑉⃗
or 𝑉⃗ = 𝑉⃗ + 𝑉⃗
Where, 𝑉⃗ = absolute velocity of aeroplane
and, 𝑉⃗ = velocity of wind.
Illustration:
An aeroplane flies along a straight path A to B and returns back again. The distance between A and
B is 𝑙 and the aeroplane maintains the constant speed v. There is a steady wind with a speed u at an
angle 𝜃 with line AB. Determine the expression for the total time of the trip.
Sol. A to B:

Velocity of plane along AB = v cos𝛼 – ucos𝜃, and for no-drift from line
AB: v sin𝛼 = usin 𝜃
⟹ sin𝛼 =
time taken from A to B :
tAB =
B to A:

velocity of plane along BA = v cos𝛼 + u cos𝜃


and for no drift from line AB : v sin𝛼 = usin𝜃
𝑢𝑠𝑖𝑛𝜃
⟹ sin𝛼 =
time taken from B to A:
tBA =

total time taken = tAB + tBA = +


𝑐𝑜𝑠𝜃

2𝑣𝑙𝑐𝑜𝑠𝛼
= = .

167
Ex. Find the time an aeroplane having velocity v, take to fly around a square with side a and the wind
blowing at a velocity u, in the two cases,
(a) if the direction of wind is along one side of the square,
(b) If the direction of wind is along one of the diagonals of the square

Ans. (a) (V+√𝑉 − 𝑢 ) (b) 2√2𝑎 .

Condition to collide or to reach at the same point


When the relative velocity of one particle w.r.t. to other particle is directed towards each other then
they will collide. (If there is a zero relative acceleration).
Illustration:Three particles A, B and C are situated at the vertices of an equilateral triangle ABC of side a at
t = 0. Each of the particles moves with constant speed v. A always has its velocity along AB, B
along BC and C along CA. At what time will the particle meet each other?
Sol. The motion of the particles is roughly sketched in figure. By symmetry they will meet at the centroid
O of the triangle. At any instant the particles will from an equilateral triangle ABC with the same.

Centroid O. All the particles will meet at the centre. Concentrate on the motion of any one particle,
say B. At any instant its velocity makes angle 30º with BO.
The component of this velocity along BO is v cos 30º. This component is the rate of decrease of the
distance BO. Initially.
/
BO = = = displacement of each particle.

Therefore, the time taken for BO to become zero
/√
= = = .
√ ×√

Alternative: Velocity of B is v along BC. The velocity of C is along CA. Its


component along BC is v cos 60º = v/2. Thus, the separation BC
decreases at the rate of approach velocity.

168
∴approach velocity = v + =
Since, the rate of approach is constant, the time taken in reducing the separation BC from a to zero

is t = 3𝑉 =

Q. Six particles situated at the corners of a regular hexagon of side a move at a constant speed v. Each
particle maintains a direction towards the particle at the next corner. Calculate the time the particles
will take to meet each other.
Ans. 2 a/v.

Q. ‘A’ moves with constant velocity u along then ‘x’ axis. B always has
velocity towards A. After how much time will B meet A if B moves
with constant speed v. What distance will be travelled by A and B.

Ans. distance travelled by A = ,

distance travelled by B =

Illustration:
Two cars A and B are moving west to east and south to north respectively along crossroads. A
moves with a speed of 72 kmh–1 and is 500 m away from point of intersection of cross roads and B
moves with a speed of 54 kmh–1 and is 400 m away from point of intersection of cross roads. Find
the shortest distance between them?
Sol.
Method – I (Using the concept of relative velocity)

169
In this method we watch the velocity of A w.r.t. B. To do this we plot the resultant velocity Vr.
Since the accelerations of both the bodies is zero, so the relative acceleration between them is also
zero. Hence the relative velocity will remain constant. So the path of A with respect to B will be
straight line and along the direction of relative velocity of A with respect to B. The shortest distance
between A & B is when A is at point F (i.e. when we drop a perpendicular from B on the line of
motion of A with respect to B).

From figure
tan𝜃 = = = ........................(i)
This q is the angle made by the resultant velocity vector with the x-axis.
Also we know that from figure
OE = = .............................. (ii)
From equation (i) & (ii) we get
x = 375 m
∴ EB = OB – OE = 400 – 375 = 25 m
But the shortest distance is BF.
From magnified figure we see that BF = EB cos𝜃 = 25 ×
∴ BF = 20 m

Method II (Using the concept of maxima – minima)

A & B be are the initial positions and A’,B’ be the final positions after time t.
B is moving with a speed of 15 m/sec so it will travel a distance of BB’ = 15t during time t.
A is moving with a speed of 20 m/sec so it will travel a distance of AA’ = 20t during time t.
So
OA’ =500 – 20 t
OB’ = 400 – 15 t

170
∴ A’B’2 = OA’2 + OB’2 = (500 – 20t)2 + (400 – 15t)2 .................. (i)
For A’B’ to be minimum A’B’2 should also be minimum

( ) ( ) ( )
∴ = =0

= 2(400 – 15t) (–15) + 2(500 – 20t) (–20) = 0


= – 1200 + 45t = 2000 – 80 t
∴ 125 t = 3200
128
∴ t= s.
128
Hence A and B will be closest after s.
( )
Now comes out to be positive hence it is a minima.
On substituting the value of t in equation (i) we get
∴ A’B’2 = 400 − 15 × + 500 − 20 ×
= 16 + (12) = 20 m ∴ Minimum distance A’B’ = 20 m.

Method III (Using the concept of relative velocity of approach)

After time t let us plot the components of velocity of A & in the direction along AB. When the
distance between the two is minimum; the relative velocity of approach is zero.

∴VAcos𝛼f + VB sin𝛼f = 0
(where af is the angle made by the line A’B’ with the x-axis)
20 cos𝛼f = – 15 sin𝛼f

∴tan𝛼f = − =−
Here do not confuse this angle with the angle 𝜃 in method (I)
because that 𝜃 is the angle made by the resultant with x-axis.
Here 𝛼fis the angle made with x-axis when velocity of approach in zero,

15𝑡
∴ =−

∴ t= So, OB’ = 16 m and OA’ = – 12m

A’B’ = 16 + (−12) = 20 m

171
CLASS EXERCISE:

Qus. Two ships are 10 km apart on a line running south to north. The one farther north is steaming west at
20 km h–1. The other is steaming north at 20 km h–1. What is their distance of closest approach?
How long do they take to reach it?
Ans. 5√2 km/h; 1/4 h = 15 min consider the situation shown in figure for the two particle A and B.

Qus. (1)Will the two particles will collide


(2)Find out shortest distance between two particles
Ans. (1) The particles will not collide
(2) 4√5m.

Note: Muzzle Velocity is the velocity of bullet with respect to the gun i.e. it is Relative Velocity.

EXERCISE 1
1. A stone is thrown upwards with a velocity 50 ms–1. Another stone is simultaneously thrown
downwards from the same location with a velocity 50 ms–1. When the first stone is at the highest
point, the relative velocity of the second stone w.r.t. the first stone is:
(A) Zero (B) 50 ms–1 (C) 100 ms–1 (D) 150 ms–1
2. A thief is running away on a straight road in a jeep moving with a speed of 9 m s–1. A police man
chases him on a motor cycle moving at a speed of 10 m s–1. If the instantaneous separation of the
jeep from the motorcycle is 100m, how long will it take for the police man to catch the thief?
(A) 1s (B) 19s (C) 90s (D) 100s
3. Two cars are moving in the same direction with a speed of 30 km h–1. They are separated from each
other by 5 km. Third car moving in the opposite direction meets the two cars after an interval of 4
minutes. What is the speed of the third car?
(A) 30 km h–1 (B) 35 km h–1 (C) 40 km h–1 (D) 45 km h–1

4. Shown in the figure is the displacement time graph for two children going home from the school.
Which of the following statements about their relative motion is true?
Their relative velocity:
(A) first increases and then decreases
(B) first decreases and then increases
(C) is zero
(D) is non-zero constant.

172
5. A person standing on the escalator takes time t1 to reach the top of a tower when the escalator is
moving. He takes time t2 to reach the top of the tower when the escalator is standing. How long will
he take if he walks up a moving escalator?
(A) t2 – t1 (B) t1 + t2 (C) t1 t2 /(t1 – t2) (D) t1 t2/(t1 + t2)
6. Shown in the figure are the velocity time graphs of the two particles P1 and P2. Which of the
following statements about their relative motion is true?
Their relative velocity:
(A) is zero
(B) is non-zero but constant
(C) continuously decreases
(D) continuously increases
7. Two particles P1 and P2 are moving with velocities v1 and v2 respectively. Which of the statement
about their relative velocity vr 12 is true?
(A) vr 12> (v1 + v2) (B) vr 12 cannot be greater than v1 – v2
(C) vr 12 cannot be greater than v1 + v2 (D) vr 12< (v1 + v2)
8. Two identical trains take 3 sec to pass one another when going in the opposite direction but
only 2.5 sec if the speed of one is increased by 50 %. The time one would take to pass the other
when going in the same direction at theiroriginal speed is:
(A) 10 sec (B) 12 sec (C) 15 sec (D) 18 sec
9. Two billiard balls are rolling on a flat table. One has velocity components vx = 1m/s, vy =√3 m/s
and the other has components vx = 2m/s and vy = 2 m/s. If both the balls start moving from the same
point, the angle between their path is -
(A) 60° (B) 45° (C) 22.5° (D) 15°
10. A battalion of soldiers is ordered to swim across a river 500 ft wide. At what minimum rate should
they swim perpendicular to river flow in order to avoid being washed away by the waterfall 300 ft
downstream? The speed of current is 3 m.p.h.:
(A) 6 m.p.h. (B) 5 m.p.h. (C) 4 m.p.h. (D) 2 m.p.h.
11. A boat, which has a speed of 5 km/hr in still water, crosses a river of width 1 km along the shortest
possible path in 15 minutes. The velocity of the river water in km/hr is -
(A) 1 (B) 3 (C) 4 (D) √41
12. A bucket is placed in the open where the rain is falling vertically. If a wind begins to blow at double
the velocity of the rain, how will be rate of filling of the bucket change?
(A) Remain unchanged(B) Doubled (C) Halved (D)Become four times
13. A car with a vertical wind shield moves along in a rain storm at the speed of 40 km/hr. The rain
drops fall vertically with a terminal speed of 20 m/s. The angle with the vertical at which the rain
drop strike the wind shield is -
(A) tan–1 (5/9) (B) tan–1(9/5) (C) tan–1 (3/2) (D) tan–1(3)

173
SUBJECTIVE PROBLEMS:
14. Men are running along a road at 15 km/h behind one another at equal intervals of 20 m.
Cyclist are riding in the same direction at 25 km/h at equal intervals of 30 m. At what
speed an observer travel along the road in opposite direction so that whenever he meets
a runner he also meets a cyclist?

15. Two perpendicular rail tracks have two trains A & B respectively. Train A moves north with a speed
of 54 km h–1 and train B moves west with a speed of 72 km h–1. Assume that both trains starts from
same point. Calculate the
(a) rate of separation of the two trains
(b) relative velocity of ground with respect to B
(c) relative velocity of A with respect to B.

16. A man is swimming in a lake in a direction of 30° East of North with a speed of 5
km/hr and a cyclist is going on a road along the lake shore towards East at a speed of
10 km/hr. In what direction and with what speed would the man appear to swim to the
cyclist.

17. A motor boat has 2 throttle position on its engine. The high speed position propels the
boat at 10 km hr -1 in still water and the low position gives half the higher speed. The
boat travels from its dock downstream on a river with the throttle at low position and
returns to its dock with throttle at high
position. The return trip took 15% longer time than it did for the downstream trip. Find
the velocity of the water current in the river.

18. (I) A man can swim with a speed of 4 km h–1 in still water. How long does he take to cross a river 1
km wide if the river flows steadily at 3 km h–1 and he makes his strokes normal to the river current?
How far down the river does he go when he reaches the other bank?
(II) If he keeps himself always at an angle of 120°C with the river flow while swimming.
(a) Find the time he takes to cross the river. (b) At what point on the opposite bank will he arrive?

19. A river is flowing from west to east at a speed of 5 m/min. A man on the south bank of the river,
capable of swimming at 10 m/min in still water, wants to swim across the river in shortest distance.
In what direction should he swim?

20. An airplane is flying with velocity 50√2km/hour in north-east direction. Wind is blowing at 25
km/hr from north to south. What is the resultant displacement of airplane in 2 hours?

174
21. When a train has a speed of 10 m s–1 eastward, raindrops that are falling vertically with respect to
the earth make traces that are inclined 30º to the vertical on the windows of the train.
(a) What is the horizontal component of a drop's velocity with respect to the earth? With respect to
the train?
(b) What is the velocity of the raindrop with respect to the earth? With respect to the train?

22. To a man walking at 7 km/h due west, the wind appears to blow from the north-west, but when he
walks at 3 km/h due west, the wind appears to blow from the north. What is the actual direction of
the wind and what is its velocity?

23. When a motorist is driving with velocity 6 ̂ + 𝟖 ̂, the wind appears to come from the direction ̂.
When he doubles his velocity the wind appears to come from the direction ̂ + ̂. Then the true
velocity of the wind expressed in the form of 𝒂 ̂ + 𝒃 is ______.

24. 'n' numbers of particles are located at the vertices of a regular polygon of 'n' sides having the edge
length 'a'. They all start moving simultaneously with equal constant speed 'v' heading towards each
other all the time. How long will the particles take to collide?

25. Two ships are 10 km apart on a line running south to north. The one further north is streaming west
at 40 km/hr. The other is streaming north at 40 km/hr. What is their distance of closest approach and
how long do they take to reach it?

26. A ship is sailing towards north at a speed of√2m/s. The current is taking it towards East at the rate of
1 m/s and a sailor is climbing a vertical pole on the ship at the rate of 1 m/s. Find the velocity of the
sailor in space.

27. A motorboat is observed to travel 10 km hr–1 relative to the earth in the direction 37º north of east.
If the velocity of the boat due to the wind only is 2 km hr–1 westward and that due to the current
only is 4 km hr–1 southward, what is the magnitude and direction of the velocity of the boat due to
its own power?

28. A person P sitting on a wooden block (which does not move relative to water) in a flowing river sees
two swimmers A and B. A and B both have constant speed vm relative to water. P observes that A
starts from one point of the river bank and appears to move perpendicular to the river flow. P also
observes that B starts from some point on the other bank at the same time and moves downstream.
The width of the river is 'd' and it flows with velocity vr. If A and B both reach a point at the same
time, than find the initial separation between A and B.

175
29. A motorboat going down stream overcome a float at a point M.60 minutes later it turned back and
after some time passed the float at a distance of 6 km from the point M. Find the velocity of the
stream assuming a constant velocity for the motorboat in still water.

30. 2 swimmers start from point A on one bank of a river to reach point B on the other bank, lying
directly opposite to point A. One of them crosses the river along the straight line AB, while the other
swims at right angles to the stream and then walks the distance which he has been carried away by
the stream to get to point B. What was the velocity (assumed uniform) of his walking if both the
swimmers reached point B simultaneously? Velocity of each swimmer in still water is 2.5 km hr-1
and the stream velocity is 2 km hr-1.

31. An airplane pilot sets a compass course due west and maintains an air speed of 240 km. hr–1. After
flying for hr, he finds himself over a town that is 150 km west and 40 km south of his starting point.

(a) Find the wind velocity, in magnitude and direction.

(b) If the wind velocity were 120 km. hr–1 due south, in what direction should the pilot
set his course in order to travel due west? Take the same air speed of 240 km. hr–1.

32. Two straight AOB and COD meet each other right angles. A person walking at a speed of 5 km/hr
along AOB is at the crossing O at noon. Another person walking at the same speed along COD
reaches the crossing O at 1:30 PM. Find at what time the distance between them is least and what is
its value?

176
JEE MAIN
LEVEL –I

1] The displacement x in meters of a body varies with time t in sec. As x = 𝑡 + 16𝑡 + 2.Then the initial
velocity of the body and acceleration are
a) zero, 2 m/s2 b)− m/s16 m/s2 c) 16 m/s, − m/s2 d) 3 m/s, 4 m/s2
2] The displacement of a particle varies with time as x= - t2+16t+2. When is the velocity
of the particle zero?
a) 2 sec. b) 4 sec. c) 6 sec. d) 8 sec.
3] A body starting from rest travels a distance of 36m in the first two sec. What is the distance traveled in
the 11thsec.?
a) 160m b) 180m c) 189m d) 209m
4] A point particle starting from rest has a velocity that increases linearly with time such that v = pt where
p = 4 m/s2. The distance covered in the first 2 sec will be
a) 6 m b) 4 m c) 8 m d) 10 m
5] For a body traveling with uniform acceleration its final velocity V = √180 − 7𝑥where x is the distance
traveled by the body in meters. Then the acceleration is
a) –8 m/s2 b) –3.5 m/s2 c) –7 m/s2 d) 180 m/s2
6] An automobile travelling with a speed of 60 km/h, can brake to stop with in a distance of 20m. If the car
is going twice as fast i.e., 120 km/h, the stopping distance will be
a) 20 m b) 40m c) 60m d) 80m
7] A body is released from certain height. After falling for some time, if “g” vanishes then
a) The body continues to move with uniform acceleration
b) The body continues to move with uniform retardation
c) The body continues to move with uniform velocity
d) The body continues to move with uniform variable acceleration
8] Two stones of different masses are dropped simultaneously from the top of a building
a) Larger stone hits the ground earlier b) Smaller stone hits the ground earlier
c) Which of the stones reach the ground earlier depends on the composition of the stone
d) Both the stones reach the ground simultaneously
9] A stone dropped from the top of a tower is found to travel (5/9) of the height of the tower during the last
second of its fall. The time of its fall is
a) 2 s b) 3 s c) 4 s d) 5 s
10] A freely falling body has velocity V after falling through a distance ‘h’. The distance it has to fall
further for its velocity to be doubled is
a) 3 h b) 4 h c) 6 h d) 8 h

177
11] The height from which a body is released is numerically equal to the velocity acquired finally. Then that
height is equal to
a) g b) 2 g c) 4 g d) 8 g
12] A body projected vertically up with certain velocity reaches a maximum height of 75 m. Another body
with double the mass is thrown up with double the initial velocity. Then maximum height reached by it
would be
a) 100 m b) 150 m c) 225 m d) 300 m
13] A body projected vertically up reaches a maximum height of 9.8 m. Its time of flight is
a) 4 s b) 2 s c) √2s d) 2√2 s
14] In order to keep a body in air above the earth for ‘t’ a body is projected with certain velocity. If S is the
total distance travelled by the body during the motion, then
√ √
a) t = b) t = c) t = d) t =
15] A body is projected vertically up and the total distance travelled by it is S. Then total time of its flight
will be
√ √ √ √
a) b) c) d)
16] A body is projected vertically up with a velocity u from the ground which reaches the ground after
certain time. During this motion its average velocity and average speed are respectively
a) V and O b) O and V c) V/2 and O d) O and V/2
17] A person travels 10km North and then 10 km east in equal intervals of 1hr each. Then the displacement,
average speed and average velocity are.
18] A vehicle travels 20 km west and then 5km North in equal intervals of 1 hr each. Then the
displacement, average speed and average velocity are
19] A particle travels 3km North and 4 km East, in equal intervals of 1 hour each, the displacement, average
speed, average velocity are
20] A vehicle moves 5km East and then 12km South, in equal intervals of 1 hour each, the displacement,
average speed, average velocity are ?

178
LEVEL – II

1] A person walks a distance of 30m towards west with a speed of 2 ms-1 and 40m towards north with a
speed of 1.5 ms-1. Then average speed and average velocity for his journey
42 -1
a) ms-1, ms b) ms-1, ms-1 c) 20 ms-1, 25 ms-1 d) 25 ms-1, 20 ms-1
25
2] A body starts from rest and moves with a uniform acceleration of 10ms-2 in the first 10s. During the
next 10s it moves with the maximum velocity attained. The total displacement of the body is
a) 2000m b) 1000m c) 1500m d) 500m
3] Read the following statements and choose the correct answer.
a) For a freely falling body the average velocity is proportional to square root of height of fall.
b) For a freely falling body the displacements in successive equal time intervals are in the ratio
1:4:9
c) For a vertically projected body the displacement during last second of time of flight changes with
velocity of projection.
d) For a body projected from the top of the tower the displacement of the body is negative when the
body crosses the point of projection.
4] A body is dropped from a height of 5m. It penetrates into the sand on the ground through a distance of
10 cm before coming to rest. Then retardation of the body in the sand is
a) _490 ms---2 b) 490 ms---2 c) _ 980 ms---2 d) 980 ms---2
5] A freely falling body moves from A to B during 2 seconds and from B to C during 1 second. If
AB = BC, the distance between A and B is
a) g b) 2g c) 3g d) 4g
6] A body falling for 2 seconds covers a distance y equal to that covered in the next second. If
g = 10 ms---2, the value of y is
a) 20m b) 10m c) 30m d) 60m
7] A ball is dropped from a height 80 m above the ground. After the ball has fallen for 2seconds a second
ball is thrown straight down after it. If both hit the ground simultaneously initialvelocity of the second
ball is (g = 10 ms----2)
a) 10 ms---1 b) 20 ms---1 c) 15 ms---1 d) 30 ms---1
8] A body after falling for time‘t’ strikes a glass plate with velocity V and breaks it. After that it loses
1/4th of its velocity. After next ‘t’ seconds its velocity would be
a) 2V b) 7V/4 c) 3V/4 d) 4V/3
9] A body projected vertically from the ground with velocity u. When it crosses a point at a height h above
the ground its velocity is V. If H is the maximum height reached by it above the ground,
then =

a) (1 - V / u) b) (1 + V / u) c) (1-V2 / u2) d) (1 + V2 / u2)

179
10] A body projected vertically up with velocity from the ground reaches a maximum height h1. If the same
is projected with velocity u2 it reaches a maximum height h2, To reach a maximum height
(h1 + h2) it should be projected up with a velocity equal to
a) u1 + u2 b) u +𝑢 c) √u1 + u2 d) √u1 u2
11] A body projected vertically up from the ground crosses point A and B with velocities 2V and V
respectively. Separation between A and B is
a) V2/2g b) V2/g c) 3V2/4g d) 3V2/2g
12] A boy has projected a body vertically up with a velocity of 50 ms–1. After two seconds he projected
another body with a velocity of 100 ms–1. (g = 10 ms–2). They meet ‘t’ seconds after the first one is
projected then t =
a) 11/7 sec b) 22 sec c) 22/7 sec d) 7 sec
13] A stone is dropped from a height h. simultaneously another stone is thrown up from the
ground which reaches a height 4 h. The two stones cross each other after time
√ √
a) b) c) 8𝑔ℎ d) 2𝑔ℎ
14] A rubber ball is dropped from a height of 5m on a planet where the acceleration due to gravity is not
known. On bouncing it rises to 1.8 m. The ball loses its velocity on bouncing by a factor of
a) 16/25 b) 2/5 c) 3/5 d) 9/2
15] Ratio of the distances covered by a freely falling body in the last and last but one second of its fall is 4 :
1. The height from which it falls is
a) 9.8 m b) 19.6 m c) 44.1 m d) 98 m
16] If a body loses half of its velocity on penetrating 3 cm in a wooden block, then how much will it
penetrate more before coming to rest?
a) 1 cm b) 2 cm c) 3 cm d) 4 cm
17] Speeds of two identical cars are u and 4u at the specific instant. The ratio of the respective distances in
which the two cars are stopped from that instant is
a)1 : 1 b) 1 : 4 c) 1 : 8 d) 1 : 16
18] From a building two balls A and B are thrown such that A is thrown upwards and B downwards (both
vertically). If vA and vB are their respective velocities on reaching the ground, then
a) vB>vA b) vA = vB
c) vA>vB d) their velocities depend on their masses.
19] A car, moving with a speed of 50 km/hr can be stopped by brakes after at least 6m. If the same car is
moving at a speed of 100 km/hr, the minimum stopping distance is
a) 12m b) 18m c)24m d)6m
20] An automobile travelling with a speed of 60 km/h, can brake to stop within a distance of 20m. If the car
is going twice as fast i.e., 120 km/h, the stopping distance will be
a) 60 m b) 40 m c) 20 m d) 80 m

180
21] A ball is released from the top of a tower of height h meters. It takes T seconds to reach the ground.
What is the position of the ball at second?
8ℎ 7ℎ
a) meters from the ground b) meters from the ground

c) meters from the ground d) meters from the ground


18
22] The relation between time t and distance x is t = ax2+ bx where a and b are constants. The acceleration
is
a) 2 bv3 b) -2abv2 c) 2av2 d) -2av3
23] A car, starting from rest, accelerates at the rate f through a distance S, then continues at constant speed
for time t and then decelerates at the rate to come to rest. If the total distance traversed is 15 S, then
a) S = ft2 b) S = ft c) S = ft2 d) S = ft2
24] A parachutist after bailing out falls 50 m without friction. When parachute opens it decelerates at 2
m/s2. He reaches the ground with a speed of 3 m/s. At what height, did he bail out?
a) 182m b) 91m c) 111m d) 293m
25] A particle located at x = 0 at time t = 0, starts moving along the positive x - direction with a
velocity ‘v’ that varies as v = 𝛼√𝑥. The displacement of the particle varies with time as
a) t2 b) t c) t1/2 d) t3
26] The velocity of a particle is v = v0 + gt +ft2. If its position is x = 0 at t = 0, then its displacement after
unit time (t = 1) is
a) v0 + 2g + 3f b) v0 + g/2 + f/3 c) v0 + g + f d) v0 + g/2 +f
27] A body is at rest at x = 0, At t = 0, it starts moving in the positive x-direction with a constant
acceleration. At the same instant another body passes through x = 0 moving in the positive x-
direction with a constant speed. The position of the first body is given by x1(t) after time ‘t’ and that of
the second body by x2 (t) after the same time interval. Which of the following graphs correctly
describes (x1 - x2) as a function of time ‘t’?
(x1 - x2) (x1 - x2) (x1 - x2) (x1 - x2)

a) b) c) d)
t O t O t O t
O

28] A particle covers 1/4th of the total distance at a speed of 20 km/hr, 1/2 of the total distance at a speed of
40 km/hr, 1/6th of the total distance at a speed of 10km/hr and remaining 1/12th of the distance at a
speed of 30km/hr. What is the average speed in km/hr?
a) 22.5 km/hr b) 15 km/hr c) 30 km/hr d) 11.25 km/hr

181
JEE ADVANCED
1] The photograph of a ball is taken which is dropped down along a vertical scale from the zero division.
During the exposure the ball moved from 40 cm division to 90 cm division. Then time of exposure of
the shutter of the camera is
a) 1 / 7 s b) 2 / 7 s c) 3 / 7 s d) 4 / 7 s
2] In the arrangement shown in the figure the ends P and Q of an unstretchable string move downwards
with uniform speed U. Pulleys A and B are fixed. Mass M moves upwards with a speed:

a) 2U cos𝜃 b) U/cos𝜃 c) 2U/cos𝜃 d) U cos𝜃


3] A particle is moving eastwards with a velocity of 5 m/s In 10 s the velocity changes to 5 m/s
northwards. The average acceleration in this time is:
a) zero b) 1/√2 m2s towards north-east
c) 1/√2m2s towards north-west d) 1/2 m/s2 towards north
4] A river is flowing from west to east at a speed of 5 metre per minute. A man on the south bank of the
river, capable of swimming at 10 metre per minute in still water, wants to swim across the river in the
shortest time. He should swim in a direction :
a) due north b) 30° east of north
c) 30° west of north d) 60° east of north
5] A boat which has a speed of 5 km/hr in still water crosses a river of width 1 km along the shortest
possible path in 15 minutes. The velocity of the river water in km/hr is
a) 1 b) 3 c) 4 d) √41
6] A particle P is sliding down a frictionless hemispherical bowl. It passes the point A at t = 0. At this
instant of time, the horizontal component of its velocity is v. A boat Q of the same mass as P is ejected
from A at t = 0 along the horizontal string AB, with the speed v. Friction between the bead and the
string may be neglected. Let tP and tQ be the respective times taken by P and Q to reach the point B.
Then: [1993]

𝑙𝑒𝑛𝑔𝑡ℎ
a) tP< tQ b) tP = tQ c) tP> tQ d) =

182
7] In 1.0 s, a particle goes from point A to point B, moving in a semicircle (see figure). The magnitude of
the average velocity is:

a) 3.14 m/s b) 2.0 m/s c) 1.0 m/s d) zero


8] A ball is dropped vertically from a height d above the ground. It hits the ground and bounces up
vertically to a height d/2. Neglecting subsequent motion and air resistance, its velocity v varies with
height h above the ground as:

a) b) c) d)

9] A particle starts from rest. Its acceleration (a) versus time (t) is as shown in the figure. The maximum
speed of the particle will be:

a) 110 m/s b) 55 m/s c) 550 m/s d) 660 m/s


10] A small block slides without friction down an inclined plane starting from rest. Let sn be the distance
travelled from t = n – 1 to t = n. Then is:
2𝑛
a) b) c) d)
11] The given graph shows the variation of velocity with displacement. Which one of the graph given below
correctly represents the variation of acceleration with displacement?

a) b) c) d)

12] Body A is dropped from the top of a smooth inclined plane, to move along the plane and Body B is let
freely all from the same height. The ratio of times of descent and velocities of striking the ground are
(angle of inclination is 𝜃 with horizontal)
a) cos 𝜃: 1; 1: 1 b) 1 : cos 𝜃; 1 : 1

c) sin 𝜃 : 1; 1 : 1 d) 1 : sin 𝜃; 1 : 1

*****

183
KEY

SESSION – 1AND 2

CLASS EXERCISE
1. c 2.d 3. b 4.a 5.Ans: 3√3m

HOME EXERCISE
1. a 2.b 3. a 4. b 5. a 6.a
7.Ans:i) 4𝜋m ii)20 sin (2𝜋/5)m iii) 2ms-1 iv) 10sin (2𝜋/5) ms-1 8.d

SESSION – 3,4 AND 5

CLASS EXERCISE (A)


1. b 2. b 3.a 4.c 5.b 6.c 7.b 8.d 9.b 10.a
HOME EXERCISE (A)
1. b 2. d 3.d 4.c 5.a 6.d 7.c 8.c 9. d 10.b, c 11.b
12.a13.a 14. 15.b 16.c 17.c 18.d 19.d 20.b

SESSION – 6
CLASS EXERCISE

1. a 2. 3.Ans: i) (V0) 3/2 ii) 𝑉 4.d 5. 6. Ans:


7.Ans: O, (8/3) s; -7/2
HOME EXERCISE
2.d 3.b

SESSION –7, 8AND9


CLASS EXERCISE
1.c 2.d 3.b 4.a
HOME EXERCISE
1.b 2.c 3.b 4.a 5.b 6.d 7.c 8.a

SESSION –10, 11AND12


CLASS EXERCISE (A)
1.c 2.c 3.b 4.b 5. 6.b 7.c 8.c
HOME EXERCISE (A)
1. d 2.b 3.a 4.c 5.b 6.c 7.b 8.c 9.d 10.b

184
CLASS EXERCISE (B)
1.a 2.b 3.d 4.c 5.d 6.d 7.c
HOME EXERCISE (B)
1.b2. c 3.b 4.c 5.a

CLASS EXERCISE (C)


2.a 3.Ans: i) zero ii) 4.9m iii) -9.8ms-1 iv) 3 :1 4.c 5.d
HOME EXERCISE (C)
1. a 2.a 3.d 4.d 5.a 6.c 7.d 8. b 9.a 10. c
CLASS EXERCISE (D)
1.a 2.d 3.c 4.c 5.c 6.d
HOME EXERCISE (D)
1. a 2.d 3.a 4.d 5.b 6.d
CLASS EXERCISE (E)

1.b 2. c 3.b 4.d 5.a 6. 7.a 8.b 9.Ans: 2

10.Ans: ( 3s after B is released and 1.6 m above the ground)


HOME EXERCISE (E)
1. c 2.c 3.a 4.c

SESSION – 13,14AND 15
CLASS EXERCISE (A)
1.c 2. 3.a 4.a 5.c 6.d 7.d 8.c 9.b 10.b 11.c 12.c
HOME EXERCISE (A)
1. b 2.c 3.a 4.c 5.b 6.b 7.c 8.d 9.d 10.a
CLASS EXERCISE (B)
1.b,c 2.d 3.b 5.b 6.d 7.b 9.a 10.b 11.b
HOME EXERCISE (B)
2.a 6.c 7.b 8.c 9.a 10.d
CLASS EXERCISE (C)
1.d 2.c 3.b 4.c 5.a 6.b 7.c 8.c 9.c 10.b 11.d 12.c 13.b

HOME EXERCISE (C)


1.c 2.a 3.d 4.b 5.d 6.d 7.c 8.b 9.a 10.c

185
SESSION – 16
CLASS EXERCISE
( )
1.d 2.Ans:i) 900 ii) Tan𝜙 450 = 2 Tan𝜙 = 3 3.Ans:
𝑐𝑜𝑠𝛼
HOME EXERCISE
1.a
SESSION – 17
HOME EXERCISE
1.c 2.c 3.d 4.d 5.d 6.c 7.a 8.b 9.a 10.c

COMPREHENSION

1.b2.a 3.c 4.a 5.b 6.a 7.b 8.c

SESSION – 18, 19 & 20


OBJECTIVE PROBLEMS
1. C 2.D 3.D 4. D

5. D 6.D 7.C 8. C

9. D 10.B 11.B 12.A

13. A

SUBJECTIVE PROBLEMS

14. 5 km/h
15. (a) 25 m/s or 90 km/hr (b) 20 m/s or 72 km/hr due east (c) 25 m/s or 90 km/hr at 37°N of E
16. 30º N of W at 5√3 km/hr. 17.3 km/hr.

18. (I) 0.75 km


(II) (a) h (b) km.
√ √
19. At an angle 30° west of north.
20. 50√5 km
21. (a) 0, 10 m/s West (b) 10√3 m/s, 20 m/s

22. Coming from 5 km/hr, 53° N of E

23. (4𝚤̂ + 8𝚥̂) 24. 25. , hr


186
26. 2m/s in a direction making an angle of 60º with E, 45º with N and 60º with the vertical
27. 10√2km/hr, 45º N of E
28. √2d 29.3 km/hr
30. 3 km/hr towards B
31. (a) 100 km/hr, 37° W of S (b) 30° N of W
32. 12: 45 PM

1. a>10 m/s2 2.3.6 minute


3. (a) 𝜃 = 45º; (b) 2 m/s
JEE MAIN

LEVEL –I

1.c 2.d 3.c 4.c 5.b 6.d 7.a 8.d 9.b 10.a 11.b 12.d
13.d 14. a15.c 16.d 17. Ans:i) 10√2 NE ii) 10 kmph iii) 5√2 kmph NE
-1 -1
18.Ans:i) 5√17km, tan (4) w of N ii) 12.5kmph iii) √17 kmph; tan (4) W of N
19.Ans: i) 5km, tan-1 (3/4) Nof E ii) 3.5 kmph iii) 2.5kmph, tan-1 (3/4) N of E
20. Ans:i) 13 km, tan-1(5/12) E of S ii) 8.5 kmph iii) 6.5 kmph, tan-1(5/12) E of S

LEVEL –II

1.b 2.c 3.a 4.b 5.c 6.c 7.d 8.b 9.c 10.b 11.d 12. c 13.b 14.b
15.b16.a 17.d 18.b 19.c 20.d 21.a 22.a 24. d 25.a 26.c 27.a 28. A

JEE ADVANCED

Objective Question (Only one option)

1. 2.b 3. c 4. a 5. b 6. a 7. b 8. a 9. b 10. c 11. a 12.c

*****

187

You might also like